Задания алгебра огэ: Материалы для подготовки к ОГЭ по математике

Содержание

Как подготовиться к ОГЭ по математике 2021-2022? Инструкция

ОГЭ снова отменяют?! А вот и не надейтесь. В 2022 году девятиклассникам придется писать его, чтобы получить аттестат. Так что сейчас самое время подготовиться к ОГЭ по математике — одному из самых важных предметов на экзамене. Вперед!

Как начать готовиться к ОГЭ по математике 2021-2022?

Вы читаете прошлогодний материал, однако в 2022 году в ОГЭ не было НИКАКИХ изменений.

Изменения ОГЭ по математике 2021-2022

1. Объединены задания 8 (выражение со степенями) и 13 (выражение с ФСУ), то есть количество заданий уменьшилось на одно, однако это не повлияло на перечень проверяемых знаний и навыков.

2. Изменилась суть заданий на последовательности и прогрессии: если раньше ребятам нужно было в основном работать с формулами, то сейчас больше упор на текстовые задачи с их применением.

В связи с этим количество заданий уменьшилось с 26 до 25, максимальный балл — с 32 до 31.

Чтобы подготовиться к ОГЭ по математике 2021-2022, нужно учитывать абсолютно все изменения: и в содержании заданий, и в формулировках, и в критериях. Если этого не сделать, то можно потерять важные баллы. А ОГЭ по математике — обязательный предмет, его

нельзя не сдать. Без сданного экзамена по математике вы не получите аттестат!

Чтобы этого не произошло, я тщательно изучаю критерии и особенности ОГЭ по математике со своими учениками. Когда мы готовимся к экзамену, я учу их внимательно читать задания и оформлять ответы в строгом соответствии с критериями. А содержание моего курса полностью актуально: мы разбираем только те темы, которые точно спрашивают на экзамене. Ничего лишнего!

Так что мои ученики всегда получают свои аттестаты — с ОГЭ по математике у них не бывает проблем. Хотите, чтобы и у вас их не было? Приходите ко мне на занятия, и я научу вас решать этот экзамен без ошибок!

Зачем вообще сдавать ОГЭ?

Чтобы получить основное общее образование, аттестат и возможность продолжить обучение дальше в 10 классе или колледже.

Вообще говоря, в нашей стране у молодых людей есть ряд обязанностей, касающихся учебы. Вы должны учиться, пока не выполнится один из критериев:

  • Вам исполнилось 18;
  • Вы успешно освоили 11 классов;
  • Вы поступили в колледж и учитесь там минимум до совершеннолетия.

Причем отвечают за то, как вы эти обязанности исполняете, родители.

Если решите уходить в колледж, стоит внимательнее отнестись к экзаменам по выбору. На конкурсе в ссуз самое важное — средний балл аттестата. Но, если придется выбирать между кандидатами с одинаковым баллом, приемная комиссия возьмет абитуриента с высоким баллом по профильному ОГЭ. Но заставить вас сдавать именно его не могут: даже поступая в мед колледж, можно сдать обязательные русский с математикой, а оставшиеся предметы добрать «неподходящими» географией и общагой, например.

Напоминаем: отметка ОГЭ влияет на балл в аттестате! В документ проставят среднее арифметическое годовой оценки и результата экзамена, округленные по математическим правилам.

Кого допускают к экзамену?

Важно понимать, что в выпускном классе лучше не расслабляться, и вот почему:

К ГИА допускаются обучающиеся, не имеющие академической задолженности, в полном объеме выполнившие учебный план или индивидуальный учебный план (имеющие годовые отметки по всем учебным предметам учебного плана за IX класс не ниже удовлетворительных), а также имеющие результат «зачет» за итоговое собеседование по русскому языку. 

Приказ Минпросвещения России и Рособрнадзора от 07.11.2018 № 190/1513

То есть испытание началось уже 1 сентября — когда вы решили не лениться в этом году, сдавать все вовремя, не «запускать» подготовку к собеседованию и экзаменам.

Как попасть на ОГЭ по математике 2021-2022?

Всеми организационными моментами займется школа, но вам тоже стоит представлять примерный план действий, чтобы ничего не пропустить:

Шаг 1:

выбрать экзамены. Тут школа не помощник, решение за вами.

Шаг 2: написать заявление на участие в ГИА до 1 марта включительно.

Этот этап часто проводят централизованно, но потом продолжают собирать заявления до последнего дня. Написать можно лично или отправить в школу родителей. Чтобы все получилось — захватите паспорт!

Шаг 3: поменяйте предметы, если передумали. Но поменять предметы после 1 марта можно только по уважительной причине, которую придется подтвердить документами. Плюс надо успеть минимум за 2 недели до экзаменов.

Помните, что экзамены — это определенные правила и своя атмосфера. Мы понимаем, что читать документы с порядком проведения экзамена очень скучно, но пожалуйста, сделайте это. Так вы познакомитесь с первоисточником информации и не будете верить мифам о запрете на воду с шоколадкой или о безобидности шпаргалок.

Когда и где проходит ОГЭ по математике 2021-2022?

Экзамены проводят в три периода: досрочный, основной и дополнительный. Причем в каждом есть резервные дни, которыми пользуются, если расписание испытаний «наслоится» или если не смогли присутствовать в срок по уважительной причине.

Большинство школьников сдают ОГЭ в основной период, и он точно не начнется раньше 25 мая. Расписание меняется каждый год и публикуется в официальных источниках, например, на сайте Рособрнадзора. А еще можно подписаться на наш блог, и мы поделимся расписанием, как только оно станет известно! Обычно это происходит не позднее 1 апреля.

С местом проведения тоже все серьезно. Сам экзамен начинается в 10 утра, вас ждут поездка в другую школу, проход через металлоискатели, предъявление документов охранникам… Словом, испытаете на себе все прелести процесса, заодно и потренируетесь перед ЕГЭ.

А можно сдать ОГЭ по математике заранее?

Для тех, кто торопится, как раз и придуман досрочный период, но воспользоваться им не так-то просто. Надо:

  • успеть сдать все школьные предметы раньше одноклассников, до начала досрочных ОГЭ;
  • получить «зачет» за собеседование;
  • получить «добро» на участие от школы,;
  • иметь уважительную причину, по которой вы не можете сдавать вместе со всеми, и подтвердить ее документами.

Подать заявление можно в школе.

Зачем нужен дополнительный период?

Дополнительный период — возможность для двоечников не остаться на второй год, причем схема пересдачи довольно запутанная.

Для начала, пересдать заваленный один раз ОГЭ по математике 2021-2022 можно в резервные дни своего периода. Обычно они наступают примерно через месяц после экзамена.

Если ощущаете, что времени на подготовку вам не хватит, то можно сразу написать заявление на пересдачу в дополнительный период, который будет уже осенью.

Провалите экзамен снова — останется последний шанс:

  • Если сдавали в резерв своего периода — вас ждут на дополнительном периоде.
  • Сдавали в дополнительный — добро пожаловать в резервные дни дополнительного периода.

Вас отправят на пересдачу, только если вы завалили не больше двух предметов! Если не справились с тремя или четырьмя ОГЭ, раньше следующего года уже не сдать.

После успешной сдачи экзаменов в дополнительный период вы сможете продолжить учебу в школе или поискать колледжи, готовые зачислить вас после начала учебного года.

Что будет, если не сдать ОГЭ?

Если провалитесь на всех пересдачах, то есть несколько путей:

  • Остаться на второй год,
  • Выпуститься из 9 класса со справкой без аттестата.

Второй вариант мы не советуем, потому что звучит совсем печально: без документа об образовании даже в колледж не возьмут. Только если поискать редкие программы на основе 8-ми классов, которые вы все же закончили. Ну или готовиться к ОГЭ самостоятельно, чтобы все равно пересдать с следующем году. Поэтому постарайтесь подготовиться к ОГЭ по математике 2021-2022: получить минимальный балл не так трудно, как может показаться!

Какой минимум надо набрать, чтобы сдать ОГЭ по математике 2021-2022?

Чтобы просто «отстреляться», достаточно набрать всего 8 первичных баллов из 32. Правда, с оговоркой: хотя бы 2 из них должны быть по геометрии (задания 16–20, 24–26).

Полная шкала перевода для математики в 2020 была такой. В следующем году она может слегка измениться

Еще Рособрнадзор обозначил рекомендации по минимальным баллам для приема в профильные классы:

Так как это только рекомендации, ваша школа может установить пороги на свое усмотрение

В интернете ходят жуткие истории о том, как школа просит уйти после 9 класса ребят с недостаточно высоким баллом ОГЭ. Надо понимать, что школа не может вас выгнать как минимум из-за прописки. Мы уже писали выше, что в России люди учатся до 18 лет. Единственный критерий продолжения обучения — освоение предыдущей ступени образования.

Даже если у вас одни тройки, по закону вы имеете полне право учиться дальше. Если по какому-то предмету двойка — исправляйте или оставайтесь на второй год. Выкинуть на улицу вас не имеют права вплоть до совершеннолетия.

Из каких частей состоит ОГЭ по математике?

Экзамен состоит из двух разделов: задания с коротким и задания с развернутым ответами.

Часть 1: задания с коротким ответом

Это те задачки, где надо выписать в бланк только конечные ответы.

  • Задачи 1-5 — реальная математика

Блок задач, который проверяет навыки применения математических знаний в реальной жизни. Придется прочитать длинный текст, понять его, правильно выделить значимые для решения данные (обычно их в 2-3 раза меньше, чем текста всего) и правильно их применить для ответов на задания. Весьма полезные навыки. Подробнее о том, как решать этот раздел на максимум, читайте в нашем разборе!

  • Второй раздел — привычная алгебра, задания 6-15

Вот кусочек документа с сайта ФИПИ: тут расписано, какой номер какие навыки проверяет. Документ называется «Спецификация» и лежит в открытом доступе в интернете

  • Задания 16-20 — часть, посвященная геометрии

Тут спецификация не отличается разнообразием, и каждое задание нацелено на проверку знаний о плоских геометрических фигурах, векторах и координатах.

Часть 2: задания с развернутым ответом

Тут проверяющий эксперт будет придираться к каждому значку в вашем решении, которые вам предстоит полностью расписать на бланках.

  • Задания 21-23 — снова алгебра, только посложнее,
  • Задания 24-26 — сложные задачи по геометрии.

В 2020 году ОГЭ по математике сильно поменялось. Составители собрали в кучку все задания раздела «реальная математика» и придали им новую форму. Теперь это довольно интересный блок задач 1-5. С одной стороны, вам повезло: учителя уже готовили ребят к нему целый год, и у них появился хоть какой-то опыт. С другой стороны, ОГЭ отменили, и «обкатки» так и не случилось, статистики выполнения и рекомендаций от ФИПИ по новым заданиям нет. Вы все еще первопроходцы.

Когда и как начинать подготовку к экзамену?

По статистике самые высокие результаты показывают те ребята, которым хватает мотивации заниматься постоянно по чуть-чуть, без надрыва и нервов. Конечно, начать стоит как можно раньше, особенно если есть пробелы.

Сейчас стоит сосредоточиться на базовых вещах: навыках счета, знаниях элементарной алгебры, развитии логики и понимании условий задач. Чтобы подготовиться к ОГЭ по математике 2021-2022, постарайтесь не возненавидеть ее раньше времени! Превратите подготовку в приятный досуг.

  • Обзаведитесь тренажером для устного счета. Его легко выбрать в Play Market и Apple Store
  • Порешайте задачки на смекалку и логику. Так вы научитесь воспринимать условия текстовых задач и стройно мыслить
  • Потренируйте внимательность и усидчивость. На помощь придут приложения на телефоне или… ваши увлечения! Трудно вышивать крестиком или играть на гитаре, если не хватает терпения
  • Убедитесь, что без ошибок выполняете действия с дробями, знаете формулы сокращенного умножения и свойства умножения! Серьезно, девятиклассник, который косячит в этих темах, вызывает у учителей и репетиторов приступы отчаяния.

ОГЭ по математике — не самый простой экзамен. В 11 классе школьникам дают возможность выбрать между простой базой и более углубленной профильной математикой. В 9 классе такого выбора нет, все проходят испытание на уровне профильного экзамена!

Советы от команды преподавателей MAXIMUM Education

1. Пожалуйста, забудьте о прорешивании вариантов ОГЭ из сборников!

Часто подготовка в школе сводится именно к такому алгоритму, но он малоэффективен. Вы никогда не будете уверены, что сборник с парой десятков вариантов, минимум половина из которых одинаковые, охватил достаточное количество заданий. В итоге на экзамене вас может ждать неприятный сюрприз в виде абсолютно незнакомого номера.

Гораздо эффективнее можно подготовиться к ОГЭ по математике 2021-2022, опираясь на темы. Например, девятый номер проверяет умение решать уравнения, неравенства и их системы. Уравнения могут быть четырех видов: линейные, квадратные, рациональные и иррациональные.

Взяли блок теории по линейным уравнениям — выучили — отработали на банке заданий — перешли к квадратным и т.д. Так от вас точно не ускользнет ни одна тема. В MAXIMUM Education я готовлю к ОГЭ по математике именно так.

2. Уделите время знакомству с бланками

Это дело тоже часто оставляют на последний момент — и в итоге косячат в самых обидных местах, теряя драгоценные баллы.

Идеальный вариант — отработать навык в «боевых» условиях: запланируйте 2-3 пробника! Один для вас точно организует школа. Остальные придется устроить самостоятельно.

Мои ученики во время подготовки к ОГЭ обязательно пишут пробный экзамен в «суровых» условиях, чтобы привыкнуть к напряженной атмосфере экзамена и отработать стратегию поведения.

3. Отработайте стратегию поведения на экзамене

Чтобы подготовиться к ОГЭ по математике, важно заранее подумать, какие задания и в каком порядке вы будете решать. Универсального рецепта нет: один быстро утомляется, другой — нервничает, если не закрыл самые простые задачки. А пробники — как раз способ порепетировать.

4. Готовьтесь по максимуму, если математика будет нужна для поступления

Чтобы получить хорошую отметку в аттестат, достаточно 15 первичных баллов! Но если вы уже сейчас понимаете, что для поступления в вуз понадобится математика, старайтесь подготовиться к ОГЭ математике, уделяя внимание всем заданиям. Это будет хорошей основой для ЕГЭ в 11 классе.

И помните, что без тщательной подготовки к ОГЭ по математике высокий балл не получить. Поэтому со своими учениками я прорабатываю все темы и даю им эффективные алгоритимы решения каждого задания. И экзамен они пишут стабильно на высокий балл. Записывайтесь на мои занятия, и в вашем аттестате тоже будет красоваться пятерка за математику 😎

рабочая программа » Подготовка к ОГЭ по математике » | Рабочая программа по алгебре (9 класс) на тему:

Муниципальное  общеобразовательное учреждение

«Cредняя  школа № 16»

Рассмотрено

на заседании МО школы

Протокол _____

от «__»______2016  г.

Согласовано

Зам. директора по УВР

_______________

от «____»________2016   г.

Утверждаю  директор МОУ  CШ № 16

__________М.В. Антонова

Приказ  № ____

от «__»______2016   г.

РАБОЧАЯ  ПРОГРАММА

 «ПОДГОТОВКА  К  ОГЭ   ПО  МАТЕМАТИКЕ»

Составитель: М.В Фирсова

учитель  математики

                                                                                       

г. Димитровград

2015 – 2016

Пояснительная записка.

Рабочая программа составлена на основе Федерального государственного образовательного стандарта.

Математическое образование является обязательной и неотъемлемой частью общего образования на всех ступенях школы.

Введение государственной итоговой аттестации по  математике в новой форме  в 9 классе вызывает необходимость изменения в методах и формах работы учителя. Данная необходимость обусловлена тем, что изменились требования к знаниям, умениям и навыкам учащихся в материалах экзамена по математике.

Само содержание образования существенно не изменилось, но в рамках реализации ФГОС изменилась формулировка вопросов: вопросы стали нестандартными, задаются в косвенной форме, ответ на вопрос требует детального анализа задачи. И это всё в первой части экзамена, которая предусматривает обязательный уровень знаний. Содержание задач изобилует математическими тонкостями, на отработку которых в общеобразовательной программе не отводится достаточное количество часов.

 В обязательную часть включаются задачи, которые либо изучались давно, либо на их изучение отводилось малое количество времени (проценты, стандартный вид числа, свойства числовых неравенств, задачи по статистике, чтение графиков функций), а также задачи, требующие знаний по другим предметам, например, по физике.

Основные цели курса

  • диагностика проблемных зон;
  • эффективное выстраивание систематического повторения;
  • помочь приобрести опыт решения разнообразного класса задач курса, в том числе, требующих поиска путей и способов решения, грамотного изложения своих мыслей в формате работ ОГЭ.
  • успешно пройти ГИА по математике.

Задачи курса

  • повторить и закрепить знания, умения и навыки, полученные в 5-8 и 9 классах;
  • развить способность самоконтроля: времени, поиска ошибок в планируемых проблемных заданиях;
  • сформировать спокойное, уравновешенное отношение к экзамену;
  • вести планомерную подготовку к экзамену;
  • закрепить математические знания, которые пригодятся в обычной жизни и при продолжении образования.

Место курса в учебном плане

Программой отводится  — 34 часа (1 час – в неделю )

Планируемые результаты

Ученик:

  • научится: выполнять задания в формате обязательного государственного экзамена, осуществлять диагностику проблемных зон и коррекцию допущенных ошибок, повышать общематематическую компетентность сначала в классе, в группе, затем самостоятельно;
  • получит возможность: успешно подготовиться к экзамену, самостоятельно выстраивать тактику подготовки к экзаменам с использованием материалов разных ресурсов.

Формы организации деятельности обучающихся:

  1. Групповые;
  2. Индивидуально — групповые;
  3. Компьютерные практикумы ( дома )

  Система работы по подготовке к ОГЭ по математике в 9 классе.

  1. Включать в изучение текущего учебного материала задания, соответствующие экзаменационным заданиям.
  2. В содержание текущего контроля включать экзаменационные задачи.
  3. Изменить систему контроля над уровнем знаний учащихся по математике.
  4. Итоговое повторение построить исключительно на отработке умений и навыков, требующихся для получения положительной отметки на экзамене.
  5. Подготовка ко второй части работы осуществляется как на уроках, так и во внеурочное время. Используется сборники для подготовки к экзаменам, рекомендованные ФИПИ, РАО и др.
  6. Важным условием успешной подготовки к экзаменам является тщательность в отслеживании результатов учеников по всем темам и в своевременной коррекции уровня усвоения учебного материала.

Главные изменения в ОГЭ 2016.

Математика — содержательных изменений нет.

Скорректирована система оценивания заданий 22, 23, 25, 26 (максимальный балл за выполнение каждого из них – 2). Максимальный первичный балл за выполнение всей работы снижен с 38 до 32.

План работы по подготовке учащихся к ОГЭ  по математике.

№ п/п

Мероприятия

Сроки проведения

1.

Психологическая подготовка к ГИА.

Индивидуальное консультирование учащихся.

Проведение групповых занятий  для учащихся.

В течение года,

ежедневно

В течение года,

среда 13.05ч.

(7 урок – 8 класс),

 четверг 13.05ч.

(7урок – 9 класс).

2.

Использование современных образовательных технологий, новых форм организации учебно- воспитательного процесса, способствующих повышению качества подготовки школьников к итоговой аттестации.

В течение года

3.

Пополнение классной библиотеки методической и информационной литературой, в том числе с интернет ресурсов, по подготовке к ГИА.

В течение года

4.

Беседа с учащимися: «Подготовка к ГИА  по математике: от устранения пробелов в знаниях до итоговой аттестации» (с целью выработки оптимальной стратегии подготовки к ГИА по математике).

1 четверть

5.

Регулярное проведение или присутствие на классных родительских собраний с докладами на темы:

«Ознакомление с нормативными документами по подготовке к проведению новой формы аттестации

 9-тиклассников»,«Нормативные документы по ГИА в новом учебном  году»,«Построение режима дня во время подготовки к экзаменам с учётом индивидуальных особенностей ребенка»,«Цели и технологии проведения ОГЭ в IX классе».

В течение года, по согласованию с кл.рук.

6.

Беседа с учащимися:  « Знакомство с Положением о порядке проведения государственной (итоговой) аттестации».

2 четверть

7.

Работа с учащимися:  «Работа с бланками: типичные ошибки при заполнении бланков»

(обучение работе с КИМами), практические занятия  по заполнению бланков ответов.

В течение года

8.

Работа с учащимися:

— анализ типичных ошибок учащихся при сдаче ОГЭ.

— выбор оптимальной стратегии выполнеия заданий ОГЭ

( помощь в выработке индивидуального способа деятельности в процессе выполнения экзаменационных заданий).

В течение года, среда ( 8 кл), четверг(9кл) 7урок.

9

Разбор заданий демонстрационного варианта экзамена по математике (ОГЭ-2015)

1 -2четверть

10

Подготовка, оформление информационного стенда «Подготовка к ОГЭ».

3 четверть

11

Индивидуальные консультации родителей

В течение года,

12

Работа с заданиями различной сложности.

Практикум по решению заданий второй части экзаменационной работы

Индивид работа в течение года,

16

Регулярное участие в диагностических работах.

В течение года

17

Регулярное участие в тренировочных работах.

В течение года

18

Мониторинг качества подготовки учащихся к ОГЭ.

В течение года

КАЛЕНДАРНО-ТЕМАТИЧЕСКОЕ ПЛАНИРОВАНИЕ

№ урок

№ ур  

Тема

Виды учебной деятельности

в классе

Домашнее задание

Дата проведен занят

Числа и вычисления  (6 часов) 

   

1

1

Натуральные числа. Десятичная система счисления. Признаки делимости,  деление с остатком.

Работа с демонстрационным вариантом.

Работа с открытым банком заданий.

Выполнить тест онлайн на сайте ФИПИ

2

2

Дроби. Основное свойство дроби, действия с дробями.

В-1 из сборника

Тематические тесты (сборник ОГЭ 2016)

3

3

Дроби. Задачи повышенной сложности.

Работа с демонстрационным вариантом.

Работа с открытым банком заданий.

http://mathgia.ru/

Задания по уровням (сборник ОГЭ 2016)

4

4

Рациональные числа.

Законы арифметических действий. Степень с целым показателем.

Использование скобок.

http://ege.yandex.ru/mathematics-gia/,

Пробные  варианты ОГЭ (сборник от ФИПИ, 2016)

5

5

Действительные числа. Корень третьей степени. Запись корня в виде степени.

alexlarin.net генератор заданий ОГЭ

(сборник ОГЭ 2016)

6

6

Измерения, приближения, оценки.

Зависимость между величинами, преобразования. Формулы. Зависимости прямо — и обратно пропорциональные. Прикидка и оценка результата.

В-2 из сборника

Задания из модуля «Алгебра»  открытый банк заданий ФИПИ

Алгебраические выражения  (6 часов)

7

1

Выражения с переменными.

Работа с демонстрационным вариантом.

Работа с открытым банком заданий.

http://mathgia.ru/

Задания по уровням на сайте ФИПИ

8

2

Степень с целым показателем.

Таблица степеней простых чисел.

Стандартный вид числа.

Работа с демонстрационным вариантом.

Работа с открытым банком заданий.

http://ege.yandex.ru/

Пробные варианты ОГЭ на сайте открытый банк заданий ФИПИ

9

    3

Многочлены. Преобразования, три способа разложения на множители.

alexlarin.net

Выполнить тест Uztest.ru

10

4

Многочлены. Преобразования, замена переменной. Степень и корень многочлена с одной переменной.

В-3 из сборника

Типовые варианты от ФИПИ  сб И. В. Ященко, 2015)

11

5

Алгебраическая дробь. Алгоритм тождественных преобразований выражений .

alexlarin.net генератор заданий ОГЭ

Выполнить тренинг Uztest.ru

12

6

Алгебраическая дробь. Уравнение с дробями.  Применение свойств квадратных корней. Сокращение дробей.

Работа с демонстрационным вариантом.

Работа с открытым банком заданий.

http://mathgia.ru/

Отработка стратегии поиска ошибок

 Уравнения  (6 часов)

13

1

Линейные и квадратные уравнения

Способы решения уравнений.

Корень уравнения, самопроверка.

http://ege.yandex.ru/mathematics-gia/,

Пробные варианты ОГЭ

14

2

Дробно-рациональные уравнения. Методы введения новой переменной, разложения на множители.

alexlarin.net

генератор заданий ОГЭ

Выполнить тест Uztest.ru

15

3

Системы уравнений.  Три способа решения.  Корни  уравнения.

В-4 из сборника

Типовые экзаменацион варианты

16

4

Неравенства. Числовые неравенства, их свойства. Решение неравенств.

Uztest.ru

Выполнить тренинг

17

5

Неравенства. Задания повышенной сложности.

http://ege.yandex.ru/mathematics-gia/,

Пробные варианты ОГЭ

18

6

Текстовые задачи. Решение задач с помощью уравнений  и арифметическим способом.

Uztest.ru

Тренажер с отработкой ошибок (тематические тесты)

Числовые последовательности  (1час)

19

1

Арифметическая и геометрическая прогрессии.

http://mathgia.ru/

Задания с разбором (видео разбор)

Функции  (2 часа)

20

1

Числовые функции. Элементарные функции школьного курса, их свойства и графики.

http://ege.yandex.ru/

Пробные варианты ОГЭ 

21

2

Числовые функции. Алгоритм решения задач графическим способом

alexlarin.net

Выполнить тест Uztest.ru

Координаты на прямой и плоскости  (2 часа)

22

1

Координатная прямая, плоскость.

Изображение точек.

В-5 из сборника

Типовые экзаменационные варианты сайт Гущин, Ларин.

23

2

Декартовы координаты на плоскости. Координаты середины отрезка, длина отрезка. Угол между прямыми. Угловой коэффициент.

http://mathgia.ru/

Задания по уровням с отработкой поиска ошибок

 Геометрия школьного курса  (7 часов)

 24

1

Геометрические фигуры, их свойства. Измерение геометрических величин. Начальные понятия геометрии. Движение на плоскости.

alexlarin.net

Отработка стратегии поиска ошибок Uztest.ru

  25

2

Треугольник: виды, свойства, формулы. Опорные таблицы.

В-6 из сборника

Типовые экзаменварианты

26

3

Треугольник: решение, подобные треугольники. Теоремы косинусов и синусов. Система самопроверки.

http://ege.yandex.ru/

Пробные варианты ОГЭ  

27

4

Многоугольники. Свойства многоугольников. Вычисление площадей многоугольников.

Uztest.ru

Выполнить тест по уровню сложности.

28

5

Окружность и круг.

В-7 из сборника

Открытый банк заданий ФИПИ задания второй части.

29

6

Решение задач повышенной сложности по геометрии.

Uztest.ru

Выполнить тест на сайте resugia.ru.

30

7

Векторы на плоскости.

В-8 из сборника

Типовые варианты aleklarin.ru.

Теория вероятностей  (3 часа)

31

     1

Описательная статистика

В-9- сборник

Решить вариант на сайте  aleklarin.ru.

32  

2

Теория вероятностей и комбинаторика.

В-10 сборник

Решить вариант aleklarin.ru.

33

3

Решение задач по теории вероятности.

alexlarin.net

Распечатка тестов по уровню сложности.

34

1

Итоговое занятие.

Тренировочное тестирование

С 2016 года выпускники девятых классов должны сдавать четыре экзамена формата ОГЭ, два из которых обязательные, а два по выбору. 

Баллы, полученные за верно выполненные задания, суммируются.

Для успешного прохождения итоговой аттестации необходимо набрать в сумме не менее 8 баллов, из них

-не менее 3 баллов в модуле «Алгебра»,

-не менее 2 баллов в модуле «Геометрия»

-не менее 2 баллов в модуле «Реальная математика».

За каждое правильно выполненное задание части 1 выставляется 1 балл. В каждом модуле части 2 задания оцениваются в 2 балла.

Всего заданий 26, из них:

заданий по алгебре — 11,

по геометрии — 8,

по реальной математике — 7.

Заданий базового уровня сложности 20, повышенного — 4, высокого — 2.
Работа рассчитана на 235 минут.

ЧТО МОЖНО ВЗЯТЬ С СОБОЙ НА ЭКЗАМЕН.

Таблицу квадратов двузначных чисел, формулы корней квадратного уравнения, разложения на множители квадратного трехчлена, формулы n-го члена и суммы n первых членов арифметической и геометрической прогрессий. Калькуляторы на экзамене не используются. На экзамене запрещено пользоваться мобильными телефонами и иными средствами связи.

                Проверяемые элементы содержания и виды деятельности.

Обозначение уровня сложности задания: Б — базовый, П — повышенный.

Проверяемые элементы содержания и виды деятельности

Уровень сложности задания

Максимальный балл за выполнение задания

Примерное время выполнения задания (мин.)

Задание 1. Уметь выполнять вычисления и преобразования

Б

1

2-3

Задание 2. Уметь выполнять вычисления и преобразования

Б

1

2-3

Задание 3.Уметь выполнять вычисления и преобразования, уметь выполнять преобразования алгебраических выражений

Б

1

2-3

Задание 4. Уметь решать уравнения, неравенства и их системы

Б

1

2-3

Задание 5. Уметь строить и читать графики функций

Б

1

2-3

Задание 6. Уметь строить и читать графики функций

Б

1

3-5

Задание 7. Уметь выполнять преобразования алгебраических выражений

Б

1

3-5

Задание 8. Уметь решать уравнения, неравенства и их системы

Б

1

2-3

Задание 9. Уметь выполнять действия с геометрическими фигурами, координатами и векторами

Б

1

5

Задание 10. Уметь выполнять действия с геометрическими фигурами, координатами и векторами

Б

1

5

Задание 11. Уметь выполнять действия с геометрическими фигурами, координатами и векторами

Б

1

5

Задание 12. Уметь выполнять действия с геометрическими фигурами, координатами и векторами

Б

1

5

Задание 13. Проводить доказательные рассуждения при решении задач, оценивать логическую правильность рассуждений, распознавать ошибочные заключения

Б

1

5

Задание 14. Пользоваться основными единицами длины, массы, времени, скорости, площади, объёма; выражать более крупные единицы через более мелкие и наоборот.

Б

1

5

Задание 15. Описывать с помощью функций различные реальные зависимостмежду величинами; интерпретировать графики реальных зависимостей

Б

1

5

Задание 16. Решать несложные практические расчетные задачи; решать задачи, связанные с отношением, пропорциональностью величин, дробями, процентами; пользоваться оценкой и прикидкой при практических расчетах; интерпретировать результаты решения задач с учётом ограничений, связанных с реальными свойствами рассматриваемых объектов

Б

1

5

Задание 17. Описывать реальные ситуации на языке геометрии, исследовать построенные модели с использованием геометрических понятий и теорем, решать практические задачи, связанные с нахождением геометрических величин

Б

1

10

Задание 18. Анализировать реальные числовые данные, представленные в таблицах, на диаграммах, графиках

Б

1

10

Задание 19. Решать практические задачи, требующие систематического перебора вариантов; сравнивать шансы наступления случайных событий, оценивать вероятности случайного события, сопоставлять и исследовать модели реальной ситуацией с использованием аппарата вероятности и статистики

Б

1

10

Задание 20. Осуществлять практические расчеты по формулам, составлять несложные формулы, выражающие зависимости между величинами

Б

1

10

Задание 21 (С1). Уметь выполнять преобразования алгебраических выражений, решать уравнения, неравенства и их системы, строить и читать графики функций

П

2

15-20

Задание 22 (С2). Уметь выполнять преобразования алгебраических выражений, решать уравнения, неравенства и их системы, строить и читать графики функций, строить и исследовать простейшие математические модели

П

2

15-20

Задание 23 (С3). Уметь выполнять преобразования алгебраических выражений, решать уравнения, неравенства и их системы, строить и читать графики функций, строить и исследовать простейшие математические модели

В

2

15-20

Задание 24 (С4). Уметь выполнять действия с геометрическими фигурами, координатами и векторами

П

2

15-20

Задание 25 (С5). Проводить доказательные рассуждения при решении задач, оценивать логическую правильность рассуждений, распознавать ошибочные заключения

П

2

15-20

Задание 26 (С6). Уметь выполнять действия с геометрическими фигурами, координатами и векторами

В

2

15-20

ШКАЛА ПЕРЕВОДА ОТМЕТОК

Шкала пересчета суммарного балла за выполнение
экзаменационной работы в целом в отметку по математике

Отметка по пятибалльной шкале

«2»

«3»

«4»

«5»

Суммарный балл за работу в целом

0 – 7

8 – 14

15 – 21

22 – 32

Шкала пересчета суммарного балла за выполнение
заданий, относящихся к разделу «Алгебра» в отметку по алгебре

Отметка по пятибалльной шкале

«2»

«3»

«4»

«5»

Суммарный балл за работу в целом

0 – 4

5 – 10

11 – 15

16 – 20

Шкала пересчета суммарного балла за выполнение
заданий, относящихся к разделу «Геометрия» в отметку по геометрии

Отметка по пятибалльной шкале

«2»

«3»

«4»

«5»

Суммарный балл за работу в целом

0 – 2

3 – 4

5 – 7

8 – 12

Материально-техническое обеспечение, интернет-ресурсы

1.  Компьютер, ноутбуки, интерактивная доска.

2.  Тренинг Яндекс — ОГЭ http://ege.yandex.ru/mathematics-gia/,

      модули специализированных уроков по алгебре;

3.  Тесты и тренинги на uztest.ru;

4.  Открытый банк заданий по математике http://mathgia.ru/or/gia12/Main.html

5.   Генератор вариантов ГИА-2014 и ОГЭ-2015 http://alexlarin.net/

6.  Видеоуроки по математике Кирилла и Мифодия.

Сайты для подготовки к ОГЭ и  ЕГЭ по математике.

http://fipi.ru/view/sections/211/docs/471.html — демо-версия

http://alexlarin.net — различные материалы для подготовки

http://www.egetrener.ru — видеоуроки

http://www.mathege.ru — открытый банк заданий

http://live.mephist.ru/?mid=1255348015#comments — Открытый банк

http://reshuege.ru/

http://matematika.egepedia.ru

http://www.mathedu.ru

http://www.ege-trener.ru

http://egeent.narod.ru/matematika/online/

http://alexlarin.net/ege/2010/zadc3.pdf — Подготовка к C3

http://alexlarin.net/ege/2010/C4agk.pdf — Подготовка к С4

http://alexlarin.net/ege/2010/c1c3sta.pdf — Задания С1, С3

http://vkontakte.ru/app1841458 — приложение ВКонтакте — отработка части В

http://matematika-ege.ru

http://uztest.ru/

http://www.diary.ru/~eek — Математическое сообщество.

Видео-уроки по математике.

http://egefun.ru/test-po-matematike

http://www.webmath.ru/

http://www.shevkin.ru/?action=Page&ID=752 разбор заданий С6

http://www.youtube.com/user/wanttoknowru канал с разборами всех заданий

http://www.pm298.ru/ справочник математических формул

http://www.uztest.ru/abstracts/?idabstract=18 квадратичная функция: примеры и задачи

http://www.bymath.net/ элементарная математика

http://dvoika.net/ лекции

http://www.slideboom.com/people/lsvirina презентации по темам

http://www.ph5s.ru/book_ab_mat_zad.html книги

http://uniquation.ru/ru/ формулы

http://www.mathnet.spb.ru/texts.htm методические материалы.

Литература

  1. И.В. Ященко, С.А.Шестаков. Сборник ОГЭ 2015: «Типовые тестовые задания» от разработчиков ФИПИ. Изд. «Экзамен», М.2015г.
  2. Жохов В. И., Крайнева Л. Б. Уроки алгебры 9 класс. – М.: Просвещение, 2008.
  3. Жохов В. И., Макарычев Ю. Н., Миндюк Н. Г. Дидактические материалы по алгебре, 9 класс. – М.: Просвещение, 2000.
  4. Мкарычев Ю.Н . Изучение алгебры в 7-9 классах. –М.: Просвещение, 2012.

Я сдам ОГЭ! Математика. Типовые задания. Алгебра. Геометрия (2018) скачай бесплатно



Пособие состоит из двух частей и включает три модуля: «Алгебра», «Геометрия», «Задачи с практическим содержанием». Задания каждого модуля делятся на отдельные темы (пункты) и наряду с основным блоком задач содержат задачи на повторение по другим темам. Пособия «Я сдам ОГЭ!» будут полезны учащимся 8-9 классов, преследующим разные цели: от преодоления минимального порога баллов до получения высоких баллов, а также педагогам для групповой и индивидуальной подготовкой обучающихся к ОГЭ.

Содержание
Часть 1: Алгебра
Часть 2: Геометрия

Об изменениях в ОГЭ по математике:

В 2018 г. структура Основного государственного экзамена (ОГЭ) по математике будет отлична от структуры прошлых лет. Изменения коснулись именно структуры экзамена, число заданий осталось прежним — 26 задач. Все задания модуля «Реальная математика» , за исключением геометрической задачи с практическим содержанием, объединены с заданиями модуля «Алгебра», геометрическая задача включена в модуль «Геометрия» . Тем самым, модуль «Алгебра» содержит 17 заданий, 3 из которых являются задачами повышенного и высокого уровней сложности и включены во вторую часть экзаменационной работы. Модуль «Геометрия» теперь состоит из 9 заданий, 3 из которых являются задачами повышенного и высокого уровней сложности и включены во вторую часть экзаменационной работы. Таким образом, вторая часть экзаменационной работы не изменилась. В целях обеспечения преемственности задания с алгебраической составляющей упразднённого модуля «Реальная математика» выделены в данном пособии в отдельный модуль «Задачи с практическим содержанием» . Большинство задач пособия аналогично реальным заданиям ОГЭ по математике и адаптировано под их формат.


Год 2018
Автор: И.В. Ященко, С.А. Шестаков
Жанр: Математика
Формат: PDF
Качество: Хороший скан
Иллюстрации: Черно-белые
Страниц 193+209
Размер: 94.2 Мб

Скачать Я сдам ОГЭ! Математика. Типовые задания. Алгебра. Геометрия (2018)




ОГЭ 2021 Практико-ориентированные задания 1 — 5 (зонт)

Прочитайте внимательно текст и выполните задания 1 – 5.

Задание 1 (ОГЭ 2021 зонт)

Решение: Обозначим за Х см длину спицы. Из условия известно, что треть длины спицы и ручка зонта составляют в сумме 25 см.

Составим уравнение: 1/3Х + 6,2 = 25,

1/3Х = 18,8,

Х = 18,8 * 3,

Х = 56,4 (см) – длина спицы зонта.

Ответ: 56,4.

Задание 2 (ОГЭ 2021 зонт)

Поскольку зонт сшит из треугольников, рассуждал Петя, сумму его поверхностей можно найти как сумму площадей треугольников. Вычислите площадь поверхности зонта методом Пети, если высота каждого равнобедренного треугольника, проведенная к основанию, равна 53,1 см. Ответ дайте в квадратных сантиметрах с округлением до десятков.

Решение:

Площадь поверхности зонта состоит из 8 равных равнобедренных треугольников. Проведем высоту в треугольнике на рисунке 1. Найдем площадь одного треугольника.

S = 1/2ah = 1/2 * 38 * 53,1 = 1008,9 (кв. см).

Найдем площадь поверхности зонта.

S = 8 * 1008,9 = 8071,2 ≈ 8070 (см. кв.).

Ответ: 8070.

Задание 3 (ОГЭ 2021 зонт)

Вася предположил, что купол зонта имеет форму сферического сегмента. Вычислите радиус R сферы купола, зная, что OC = R (рис. 2). Ответ дайте в сантиметрах.

Решение:

OC = R.

Рассмотрим прямоугольный треугольник AHC. Обозначим OA = x (радиус). Тогда OH = OC – HC = x – 25, AH = 50 см (половина AB из условия).

Применим теорему Пифагора.

OA2 = OH2 + AH2,

x2 = (x – 25)2 + 502,

x2 = x 2 – 50x + 625 + 2500,

50x = 3125,

x = 62,5 (см) – радиус сферы купола зонта.

Ответ: 62,5.

Задание 4 (ОГЭ 2021 зонт)

Вася нашел площадь купола зонта как площадь поверхности сферического сегмента по формуле S = 2ПRh, где R – радиус сферы, а h – высота сегмента. Рассчитайте площадь поверхности купола способом Васи. Число П округлите до 3,14. Ответ дайте в квадратных сантиметрах с округлением до целого.

Решение: S = 2ПRh = 2 * 3,14 * 62,5 * 25 = 9812,5 ≈ 9813 (кв. см).

Ответ: 9813.

Задание 5 (ОГЭ 2021 зонт)

Рулон ткани имеет длину 35 м и ширину 80 см. На фабрике из этого рулона были вырезаны треугольные клинья для 29 зонтов, таких же, как зонт, который был у Пети и Васи. Каждый треугольник с учётом припуска на швы имеет площадь 1050 кв. см. Оставшаяся ткань пошла в обрезки. Сколько процентов ткани рулона пошло в обрезки?

Решение: 35 м = 3500 см.

Найдем площадь рулона ткани. S = 3500 * 80 = 280000 (кв. см).

Рассчитаем сколько ткани ушло на один зонт. 8 * 1050 = 8400 (кв. см).

8400 * 29 = 243600 (кв. см) – ткани нужно на пошив 29 зонтов.

280000 – 243600 = 36400 (кв. см) – обрезки.

Составим пропорцию для нахождения процентов ткани (x%), которая пошла на обрезки.

280000 – 100%

36400 – x%

Найдем неизвестный член пропорции.

х = (36400 * 100) / 280000 = 13%.

Ответ: 13.

Разбираем практико-ориентированные задания 1 – 5 ОГЭ (маркировка автомобильных шин) по математике 2021.

ОГЭ. Правила оформления задания 21.

ОГЭ.

Правила оформления задания 21. Рекомендации. Типичные ошибки

Гусева Ирина Александровна

МКОУ Куминская СОШ

Общие подходы к проверке и оценке выполнения заданий

с развернутым ответом

Требования к выполнению заданий с развернутым ответом заключаются в следующем: решение должно быть математически грамотным и полным, из него должен быть понятен ход рассуждений учащегося. Оформление решения должно обеспечивать выполнение указанных выше требований, а в остальном может быть произвольным. Не следует требовать от учащихся слишком подробных комментариев (например, описания алгоритмов). Лаконичное решение, не содержащее неверных утверждений, все выкладки которого правильны, следует рассматривать как решение без недочетов.

Если решение заданий 21–26 удовлетворяет этим требованиям, то выставляется полный балл – 2 балла за каждое задание. Если в решении допущена ошибка непринципиального характера (вычислительная, погрешность в терминологии или символике и др.), не влияющая на правильность общего хода решения (даже при неверном ответе) и позволяющая, несмотря на ее наличие, сделать вывод о владении материалом, то учащемуся засчитывается балл, на 1 меньший указанного, что и отражено в критериях оценивания заданий с развернутым ответом.

В критериях оценивания по каждому конкретному заданию второй части экзаменационной работы эти общие позиции конкретизируются и пополняются с учетом содержания задания. Критерии разработаны применительно к одному из возможных решений, а именно, к тому, которое описано в рекомендациях.

При наличии в работах учащихся других решений критерии вырабатываются предметной комиссией с учетом описанного общего подхода. Решения учащихся могут содержать недочеты, не отраженные в критериях, но которые, тем не менее, позволяют оценить результат выполнения задания положительно (со снятием одного балла). В подобных случаях решение о том, как квалифицировать такой недочет, принимает предметная комиссия.

Типичные ошибки при выполнении данного задания

ПРИМЕРЫ ЗАДАНИЙ

Оцените задание

Оцените задание

За решение выставляется 1 балл, так как оно не содержит ошибок, но разложение на множители не доведено до конца.

Оцените задание

Оцените задание

За решение выставляется 0 баллов; допущена ошибка в знаках при

группировке слагаемых.

Оцените задание

Оцените задание

Допущена ошибка вычислительного характера на последнем шаге решения. Оценка снижается на 1 балл, за решение выставляется 1 балл.

Оцените задание

Оцените задание

Допущена ошибка принципиального характера в алгоритме решения неравенства. За решение выставляется 0 баллов.

Оцените задание

Оцените задание

За решение выставляется 2 балла. Все шаги выполнены верно, получен правильный ответ

Оцените задание

Оцените задание

За решение выставляется 2 балла. Все шаги выполнены верно, получен правильный ответ.

Оцените задание

Оцените задание

Работа интересная – записан верный ответ. Но присутствуют в последних строках:

а) ошибка в вычислении корня квадратного уравнения;

б) ошибка при сложении чисел с разными знаками;

в) ошибка в формуле корней квадратного уравнения;

г) ошибка при делении чисел с разными знаками.

0 баллов

Оцените задание

Оцените задание

При нахождении корней квадратного уравнения допущена неверная запись. При наличии общей формулы для нахождения корней квадратного уравнения, записанной верно, не извлечен корень из дискриминанта, все дальнейшие вычисления (с этой ошибкой) выполнены верно. Вычислительная ошибка присутствует, с её учётом дальнейшие шаги выполнены верно.

1 балл

Оцените задание

Оцените задание

Все этапы решения присутствуют, корни в правом столбце найдены верно. Неверную запись ответа можно рассмотреть как описку.

1 балл

Оцените задание

Оцените задание

Правильно выполнены преобразования, получен верный ответ, 2 балла

Оцените задание

Оцените задание

Все этапы решения присутствуют, корни найдены верно. Неверную запись ответа можно рассмотреть как неверное владение символикой (хочется надеяться, что учащийся хотел написать фигурные скобки).

1 балл

Оцените задание

Оцените задание

1 балл

Оцените задание

Оцените задание

0 баллов

Оцените задание

Оцените задание

2 балла

Оцените задание

Оцените задание

2 балла

Спасибо за внимание!

Использованы методические материалы сайта ФИПИ и сайта Решу.ОГЭ

Средний балл по огэ. Критерии оценки ОГЭ

После проверки заданий ОГЭ по русскому языку, начальная оценка за их выполнение. За экзамен можно получить от 0 до 39 баллов. Каждое задание на экзамене оценивается определенным количеством баллов: чем сложнее задание, тем больше баллов вы можете получить за него.

После этого первичный балл конвертируется в оценку по пятибалльной шкале. Перенос баллов ОГЭ осуществляется по специальной шкале баллов.Ниже представлена ​​шкала перевода баллов ОГЭ на русский язык : начальные баллы и оценка по пятибалльной шкале.

баллов ОГЭ шкала перевода: Русский

Для поступления в профильный класс необходимо набрать не менее 31 балла на экзамене. Минимальный балл в различных учебных заведениях может отличаться от указанного, конкретное значение уточняйте в самом учебном заведении.

При определении оценки по первичному баллу также учитывается уровень грамотности обучающегося (критерии ГК1-ГК4):

  • Если экзаменующийся набрал 25-33 балла и претендует на оценку «4» «он должен иметь не менее 4 баллов по грамотности, иначе ему будет выставлена ​​оценка« 3 »;
  • Если экзаменуемый набрал 34-39 баллов и претендует на оценку «5», он должен иметь не менее 6 баллов по грамотности, в противном случае ему будет выставлена ​​оценка «4».

Критерии оценки ОГЭ для любого ученика 9 класса — как меч для Ландскнехта — вещь, о которой нужно все знать и уметь применять.

Мы уверены, что ни для кого не секрет, что главная цель каждого экзаменатора не столько доказать свою компетентность в конкретном предмете, сколько преодолеть минимальный проходной порог и набрать максимально возможный балл.

Чтобы помочь вам справиться с первой серьезной проблемой в вашей жизни, мы пересмотрели классическое представление о шкале порчи.Вдумчивое изучение критериев позволит избежать ошибок при выполнении заданий, провести вдумчивую проверку и правильно подойти к процессу подготовки.

Сколько очков нужно набрать на ОГЭ

Для перехода в десятый класс необходимо пройти минимальный порог, свидетельствующий об усвоении учащимися базового набора знаний, приобретенных в течение всего девятилетнего обучения.

В случае поступления в институт ситуация обычно выглядит несколько иначе. Большинство средних профессиональных учебных заведений принимают студентов на основании оценок в их аттестатах (так называемый конкурс аттестатов).

А вот оценки в аттестате, в свою очередь, выставляются на основе среднего арифметического между годовой оценкой и полученной на экзамене, после перевода последней на пятибалльную систему (разницу легко проверить между оценочными отметками на калькуляторе).

По этой причине тем, кто стремится сразу же получить среднее техническое образование, следует начинать подготовку к экзамену заранее, чтобы не сталкиваться с возникающими проблемами в виде пересдачи после экзамена.

Критерии оценки ОГЭ по математике

ОГЭ по математике условно делится на три части: алгебра, геометрия и практические задачи (некоторые задачи относятся к первой или второй части, но имеют характер более близкий к реальности).

Если вы обратите внимание на таблицы пересчета, то обнаружите, что для тройки достаточно получить всего восемь баллов.

Однако следует иметь в виду, что ученик не сможет получить удовлетворительную оценку, потому что за геометрические задачи необходимо набрать не менее двух баллов из восьми.

По этой причине при подготовке старайтесь уделять внимание и алгебре, и геометрии, чтобы не было затруднений.

Система выставления оценок ОГЭ по русскому языку

С этим экзаменом все несколько сложнее, чем с предыдущим. По правилам оценивания он по духу ближе к ЕГЭ, так что можно уже сейчас к нему привыкнуть.

Условно его можно разделить на полутестовую часть и письменную часть (презентацию и сочинение).

Если с первым все понятно (баллы просто выставляются в соответствии с таблицей правильных ответов), то при оценке второго вступает в силу калькулятор «gk1-gk4».

Не пугайтесь аббревиатуры. Сейчас мы все объясним. Первое «GK» о том, как вы говорите по орфографии. Второй — уровень соблюдения норм пунктуации (знаков препинания). Третий — грамматика в составе и изложении. Четвертое — соблюдение речевых норм (литературный язык, отсутствие проблем с координацией и т. Д.).

Обращая внимание на вышеуказанные моменты, у вас больше шансов получить высокий балл.

Примите во внимание: если вы набрали меньше шести баллов за выполнение заданий второй части, то пятерку вы не увидите.

Критерии оценки НГЭ по биологии

Биология относится к списку специализированных экзаменов, которые используются для отбора студентов в специализированные классы.

Чтобы получить «четверку», достаточно наскрести двадцать шесть очков.При этом Минобразования рекомендует брать тех, кто по результатам аттестации набрал тридцать три балла.

Следует понимать, что планка в данном случае является преградой. Его цель — сделать так, чтобы в специализированный класс попадали только те дети, которые готовы воспринимать новый материал на основе того, что они уже получили.

Как оценивается НГЭ в социальных исследованиях

ОГЭ по обществознанию разделена на пару частей — 25 заданий с короткими ответами и 6 заданий с подробными: здесь от студента требуется проанализировать текст и ввести все необходимые данные на отдельном листе ответов.

Чтобы с честью преодолеть 15-балльный порог (три), достаточно выполнить пятнадцать заданий из первой части.

Вам предстоит побороться за первую пятерку: нужно набрать не менее 34 очков.

Критерии оценки ОГЭ по информатике

При оценке общего государственного экзамена по информатике проверяется весь комплекс знаний, который может потребоваться для дальнейшей работы в сфере IT.

К сожалению, это не всегда совпадает со школьной программой — обратите на это внимание.

При оценке работы учитываются как теоретические знания (задания по переходу из одной системы счисления в другую и др.), Так и практические (обработка массивов и программирование).

Чтобы получить удовлетворительную оценку, наберите пять баллов из двадцати двух возможных. Однако для перевода в специализированный класс вам потребуется более 15 начальных баллов.

Очков по географии на ОГЭ

От ученика ОГЭ по географии необходимо показать полное владение курсом для 7-9 классов.

Больший упор делается на оценку картографических навыков и географии Российской Федерации, изучению которой посвящены последние два года обучения.

Для преодоления минимального порога достаточно набрать 12 баллов из 32. Минобразования, в свою очередь, советует школам при наборе профильных классов обращать внимание на учеников, набравших не менее 24.

Чтобы получить отличную оценку, нужно 27 и более баллов.

Критерии оценки ОГЭ по английскому языку

Для успешной сдачи этого экзамена вам потребуются знания в трех различных областях: разговорная речь, грамматика и словарный запас.

Первый включает прослушивание текста с отметкой необходимых ответов и устный этап, второй и третий — письменную и тестовую части.

Чтобы получить оценку «три», достаточно набрать 29 баллов из 70 возможных. Если вы хотите получить «пятерку», то вам необходимо набрать 59+ баллов в сумме.

ОГЭ баллов по физике

Максимально возможный балл по физике — 40.

Получить ее можно, если ответственно подойти к подготовке знаний в данной области естествознания и их систематизации.

Минимальный порог — 10 баллов. Этого легко добиться, выполнив только задачи упрощенной части.

Если вы хотите получить отличную оценку, то уделите больше внимания задачам на последней странице экзамена: вам нужно набрать 31+ баллов.

Очки ОГЭ и отметки по химии

Этот экзамен выделяется на фоне остальных «плавающей» шкалой оценок.

Называется он так потому, что есть два похожих, но разных в одной детали, варианта проведения — с реальным экспериментом (макс.Оценка = 40) и без нее (макс. Балл = 34).

В любом случае, чтобы получить оценку «3», вам нужно 9 начальных баллов. В случае «отлично» вы должны заработать 27+ на экзамене без эксперимента и 29+ с ним.

Проходной балл в 10 классе

Чтобы попасть в 10 класс, каждый ученик должен сдать 4 экзамена. Из них 2 обязательных экзамена (по русскому языку и математике) и 2 дополнительных, которые студент выбирает самостоятельно. Чтобы ознакомиться с минимальным количеством баллов, обратите внимание на таблицу.

Арт. Рекомендованная Министерством образования начальная оценка

Формат (приблизительный / максимальный)

Русский язык 31 + / 39
Математика 18 + / 32
Биология 33 + / 46
География 24 + / 32
Иностранный язык 56 + / 70
Информатика и ИКТ 15 + / 22
Общественные науки 30 + / 39
Физика 30 + / 40
Химия (без эксперимента) 23 + / 34
Химия (с экспериментом) 25 + / 38

Заключение

Уважаемые испытуемые и их родители! Надеемся, что наша статья помогла в вашем нелегком путешествии.Советуем быть трезвым и заранее подготовиться к экзамену. Успехов в работе и учебе!

Государственная (итоговая) аттестация выпускников основной школы по новой форме проведена в 2014 году по 14 предметам. Первичные баллы за выполнение экзаменационной работы ГИА переводятся в оценку по 5-балльной шкале. В связи с этим Федеральный институт педагогических измерений (ФИПИ) опубликовал «Рекомендации по использованию и интерпретации результатов экзаменов для государственной (итоговой) аттестации выпускников средних школ по новой форме в 2015 году» (скачать документ) .Региональным комиссиям предоставлено право изменять шкалу перевода баллов в большую или меньшую сторону по обязательным предметам.

Баллы, полученные для GIA и пересчитанные в пятибалльную систему, влияют на оценки в сертификате по соответствующему предмету. В аттестате средний показатель ставится между оценкой, полученной в GIA, и годовой по предмету. Округление производится по правилам математики, то есть 3,5 округляется до 4 и 4,5 до 5.

Выпускники могут узнать оценки за экзамен в своей школе после проверки работы и утверждения результатов.

Шкала для перевода баллов на РУССКОМ ЯЗЫКЕ

Максимальное количество баллов, которое экзаменуемый может получить за выполнение всей экзаменационной работы, составляет 39 баллов

Минимальный порог: 15 баллов

* Критерии и пояснения к оценке ГИА по русскому языку

Критерий

Расшифровка классификации

Очков

GK1.Соблюдение орфографических норм

Нет орфографических ошибок или допущено не более 1 ошибки.

Сделано 2-3 ошибки

Было допущено 4 и более ошибок

GK2. Соблюдение норм пунктуации

Ошибок пунктуации нет, или Допущено не более 2 ошибок

Сделано 3-4 ошибки

Сделано 5 и более ошибок

GK3.Соблюдение грамматических норм

Грамматические ошибки Нет, или Сделана 1 ошибка

Сделано 2 ошибки

Сделано 3 и более ошибок

GK4. Соблюдение речевых норм

Ошибок речи нет или было допущено не более 2 ошибок

Сделано 3-4 ошибки

Сделано 5 и более ошибок

Шкала для переноса баллов в MATH

Максимальный первичный балл: 38 баллов (увеличено на 5 пунктов).Из них — за модуль «Алгебра» — 17 баллов, за модуль «Геометрия» — 14 баллов, за модуль «Реальная математика» — 7 баллов.

Минимальный порог: 8 баллов (из них не менее 3 баллов в модуле «Алгебра», не менее 2 баллов в модуле «Геометрия» и не менее 2 баллов в модуле «Реальная математика»)

Преодоление этого минимального результата дает выпускнику право получить в соответствии с учебной программой учебного заведения выпускной балл по математике (если изучение математики проводилось выпускником в рамках интегрированного курса математики) или по алгебре. и геометрия.

Шкала пересчета первичного балла за выполнение экзаменационной работы в целом в оценку по математике :

Шкала преобразования первичной оценки за выполнение модуля «Алгебра» в оценку по алгебре :

Шкала пересчета первичного балла за выполнение модуля «Геометрия» в оценку по геометрии :

18 точек .

Шкала перевода баллов PHYSICS

Максимальный первичный балл: 40 кредитов (увеличено на 4 пункта)

Минимальный порог: 9 баллов

Результаты экзамена могут быть использованы при приеме учащихся в профильные классы средней школы… Ориентиром для выбора в классы профиля может быть показатель, нижняя граница которого соответствует 30 баллам .

ШКАЛА ПЕРЕНОСА ОЦЕНКИ ХИМИИ

Шкала пересчета первичного балла за экзаменационную работу без реального эксперимента
( Демо-версия ОГЭ по химии №1 )

Максимальный первичный балл: 34 балла

Минимальный порог: 9 баллов

Результаты экзамена могут быть использованы при приеме учащихся в классы общеобразовательных школ.Ориентиром для выбора в классы профиля может служить показатель, нижняя граница которого соответствует 23 баллам .

Шкала пересчета первичного балла за экзаменационную работу с реальным экспериментом
( Демо-версия ОГЭ по химии №2 )

Максимальный начальный балл за работу с реальным экспериментом : 38 баллов.

Минимальный порог: 9 баллов

Результаты экзамена могут быть использованы при приеме учащихся в классы общеобразовательных школ.Ориентиром для выбора в классы профиля может служить показатель, нижняя граница которого соответствует 25 баллам .

Шкала пересчета баллов БИОЛОГИИ

Максимальный первичный балл: 46 баллов

Минимальный порог: 13 баллов

Результаты экзамена могут быть использованы при приеме учащихся в классы общеобразовательных школ. Ориентиром для выбора в классы профиля может служить показатель, нижняя граница которого соответствует 33 баллу .

Масштаб ГЕОГРАФИИ Масштаб преобразования

Максимальный первичный балл: 32 балла

Минимальный порог: 12 баллов

Результаты экзамена могут быть использованы при приеме учащихся в классы общеобразовательных школ. Ориентиром для выбора в профильные классы может служить показатель, нижняя граница которого соответствует баллам 24 .

Шкала переноса баллов ОБЩЕСТВА

Максимальный первичный балл: 39 баллов

Минимальный порог: 15 баллов

Результаты экзамена могут быть использованы при приеме учащихся в классы общеобразовательных школ.Ориентиром для выбора в классы профиля может служить показатель, нижняя граница которого соответствует 30 баллам .

Шкала переноса баллов ИСТОРИЯ

Максимальный первичный балл: 44 балла

Минимальный порог: 13 баллов

Результаты экзамена могут быть использованы при приеме учащихся в классы общеобразовательных школ. Ориентиром для отбора в классы профиля может служить показатель, нижняя граница которого соответствует 32 баллам .

ЛИТЕРАТУРА Шкала переноса шкалы

Максимальный первичный балл: 23 балла

Минимальный порог: 7 баллов

Результаты экзамена могут быть использованы при приеме учащихся в классы общеобразовательных школ. Ориентиром для выбора в профильные классы может служить показатель, нижняя граница которого соответствует баллам 15 .

Шкала передачи баллов по ИНФОРМАТИКЕ и ИКТ

Максимальный первичный балл: 22 балла

Минимальный порог: 5 баллов

Результаты экзамена могут быть использованы при приеме учащихся в классы общеобразовательных школ.Ориентиром для выбора в профильные классы может служить показатель, нижняя граница которого соответствует баллам 15 .

МАСШТАБ ДЛЯ ПЕРЕНОСА ИНОСТРАННЫХ ЯЗЫК

(АНГЛИЙСКИЙ, НЕМЕЦКИЙ, ФРАНЦУЗСКИЙ, ИСПАНСКИЙ)

Максимальный первичный балл: 70 кредитов

Минимальный порог: 29 баллов

Результаты экзамена могут быть использованы при приеме учащихся в классы общеобразовательных школ. Ориентиром для отбора в классы профиля может служить показатель, нижняя граница которого соответствует 56 баллам.

Рекомендуемая таблица для перевода баллов ОГЭ 2018 года в оценки на русском языке опубликована на официальном сайте ФИПИ (скачать).

Максимальное количество баллов, которое может получить экзаменующийся за выполнение всей экзаменационной работы, составляет 39 баллов.

Таблица 1

Таблица марок ОГЭ на 2018 год на русском языке

Распределение баллов ОГЭ 2018 на русском языке по заданиям, отраженным в демонстрационной версии ОГЭ на русском языке в файле спецификации.

стол 2

Части работ Количество задач Максимальный балл в начальной школе Тип работы
Часть 1 1
(упражнение 1)
7
Часть 2 13
(задачи 2-14)
13 Задачи с короткими ответами
Часть 3 1
(задание 15)
9 Задание с развернутым ответом
Части 1 и 3 10 баллов за практическую грамотность и реальную точность речи
Итого 15 39

Система аттестации индивидуальных заданий и экзаменационных работ в целом

Ответ на задание 1 (сокращенное изложение) части 1 работы оценивается по специально разработанным критериям.

Максимальное количество баллов за лаконичную презентацию — 7.

За правильное выполнение каждого задания части 2 работы выпускник получает 1 балл. Нулевые баллы начисляются за неправильный ответ или его отсутствие. Максимальное количество баллов, которое может получить экзаменующийся, правильно выполнивший задания части 2 работы, — 13. Оценка ответа на задание части 3 работы проводится по специально разработанным критериям.

Максимальное количество баллов за эссе-рассуждение (альтернативное задание) — 9.Оценка практической грамотности экзаменуемого и фактической точности его письменной речи производится на основании проверки презентации и эссе в целом и составляет 10 баллов.

Максимальное количество баллов , которые экзаменующийся может получить за выполнение всей экзаменационной работы — 39.

Экзаменационные работы проверяют два эксперта. По результатам проверки эксперты самостоятельно выставляют баллы за каждый ответ заданий экзаменационной работы… В случае значительного расхождения в баллах, выставленных двумя экспертами, назначается третья проверка. Значительная разница в баллах определялась в критериях оценивания по соответствующему учебному предмету.

Третий эксперт назначается председателем предметной комиссии из числа экспертов, ранее не проверявших экзаменационную работу.

Третьему эксперту предоставляется информация о баллах, выставленных экспертами, предварительно проверившими экзаменационную работу студента.Баллы, выставленные третьим экспертом, являются окончательными. «

Расхождение в 10 и более баллов, выставленных двумя экспертами за выполнение заданий 1 и 15, считается значительным (баллы суммируются по всем позициям (критериям) оценки задания каждым экспертом: IK1 — IK3, S1K1 — S1K4, S2K1 — S2K4, S3K1 — S3K4, GK1– GK4, FC1). В этом случае третий эксперт перепроверяет пункты 1 и 15 для всех оценочных позиций.

На основании баллов, начисленных за выполнение всех заданий работы, рассчитывается общий балл, который переводится в оценку по пятибалльной шкале.

Девятиклассники озадачены не только тем, как успешно сдать GIA, но и переводом своих оценок в оценивание. Школьники привыкли к пятибалльной шкале, а понять, какая будет оценка за определенное количество баллов, можно по составленной табличке ОГЭ-2018.

Ежегодно табло меняется, потому что минимальное и максимальное количество баллов при проходном балле и задачах ОГЭ … Не может быть единой шкалы для всех предметов, ведь каждому из них свой балл.

При написании ОГЭ-2018 следует знать некоторые тонкости.

При сдаче «Математики», «Русский язык» и «Химия» недостаточно просто получить минимальный балл: необходимо соответствовать некоторым критериям. Так, например, по «Математике» нужно набрать 32 балла. Сдача составляет 8 баллов, однако, если по «Алгебре» было набрано много баллов, а по «Геометрии» — 2 балла, то экзамен будет сдан на «неудовлетворительно».

На тройку написать «русский язык» несложно, но желающие получить четверку должны набрать 25-33 балла, четыре — за грамотность, потому что поставят тройку.Если ученик хочет получить пятерку, он должен набрать шесть баллов за грамотность. Грамотность оценивается по композиции, пунктуации, красоте письма, выразительности речи, орфографии.

При сдаче «Химии» без эксперимента сложнее получить пятерку. Итак, нужно набрать 31-40 баллов, с третьей части — 5 баллов. По эксперименту — 29-38 баллов, а с третьей части — 7 баллов.

Таблица перевода баллов в оценки для ОГЭ-2018

ОГЭ-2018 на «Русском языке»:

  1. 0-14 баллов.- оценка «2»;
  2. 15-24 балла — оценка «3»;
  3. 25-33 балла — оценка «4»;
  4. 34-39 р.- оценка «5».

ОГЭ-2018 по специальности «Математика»:

  1. 0-7 баллов — оценка «2»;
  2. 8-14 баллов — оценка «3»;
  3. 15-21 балл — оценка «4»;
  4. 22-32 балла — оценка «5».

ОГЭ-2018 по физике:

  1. 0-9 баллов. — оценка «2»;
  2. 10-19 б. — оценка «3»;
  3. 20-30 б. — оценка «4»;
  4. 31-40 г.- оценка «5».

ОГЭ-2018 по «Химии» без эксперимента:

  1. 0-8 баллов. — оценка «2»;
  2. 9-17 стр. — оценка «3»;
  3. 18-26 г. — оценка «4»;
  4. 27-34 б. — оценка «5».

ОГЭ-2018 по направлению «Химия» с экспериментом:

  1. 0-8 баллов. — оценка «2»;
  2. 9-18 б. — оценка «3»;
  3. 19-28 г. — оценка «4»;
  4. 29-38 б. — оценка «5».

ОГЭ-2018 по направлению «Биология»:

  1. 0-12 баллов.- оценка «2»;
  2. 13-25 г. — оценка «3»;
  3. 26-36 б. — оценка «4»;
  4. 37-46 б. — оценка «5».

ОГЭ-2018 в разделе «География»:

  1. 0-11 баллов. — оценка «2»;
  2. 12-19 б. — оценка «3»;
  3. 20-26 г. — оценка «4»;
  4. 27-32 б. — оценка «5».

ОГЭ-2018 по направлению «Обществознание»:

  1. 0-14 баллов. — оценка «2»;
  2. 15-24 г. — оценка «3»;
  3. 25-33 б. — оценка «4»;
  4. 34-39 б.- оценка «5».

ОГЭ-2018 В истории:

  1. 0-12 баллов. — оценка «2»;
  2. 13-23 г. — оценка «3»;
  3. 24-34 б. — оценка «4»;
  4. 35-44 г. — оценка «5».

ОГЭ-2018 в литературе:

  1. 0-9 баллов. — оценка «2»;
  2. 10-17 б. — оценка «3»;
  3. 18-24 г. — оценка «4»;
  4. 25-29 г. — оценка «5».

ОГЭ-2018 по направлению «Информатика и ИКТ»:

  1. 0-4 балла.- оценка «2»;
  2. 5-11 стр. — оценка «3»;
  3. 12-17 стр. — оценка «4»;
  4. 18-22 г. — оценка «5».

ОГЭ-2018 по направлению «Иностранные языки»:

  1. 0-28 баллов. — оценка «2»;
  2. 29-45 г. — оценка «3»;
  3. 46-58 б. — оценка «4»;
  4. 59-70 б. — оценка «5».
  1. «Русский язык» — 31;
  2. «Математика» — 19 для физико-математических наук, 18 для других;
  3. Физика — 30;
  4. «Химия» — 23 без опыта, 25 — с ним;
  5. Биология — 33;
  6. «География» — 24;
  7. Обществознание — 30;
  8. «История» — 32;
  9. Литература — 19;
  10. «Информатика» — 15;
  11. «Иностранный язык» — 56.

Проходные очки легко набрать, потому что это минимум, с которого начинается тройка лучших. Пройти ОГЭ-2018 не составит труда, ведь никто не хочет, чтобы на пересдачу приходило много детей. Кроме того, чем больше пересдач, тем менее престижной будет школа, поскольку предполагается, что учителя вложили мало знаний в головы детей.

Задание 3 С использованием; The Inws Huralnithnt $ anc exixnentinls sulve tt «valtart (uual (ons jCT091 Muenc xozy » () грех () 4 грех coto7) Оге (0783-я 32-7 cortechoo 4 значащих цифры Упростите tolwILE @quation, используя только положительные индексы E MM Ialln? d оцените его, когда L = 10 мл V (t) = Васин (0Aat + WV Генерилор юл; нэ Volty Оцените следующее: Напряжение на Kenerator alter 2.5 секунд il Vs Z0 V Напряжение tirnc t = ccands к значительным fxurs Время, когда генератор впервые достигает максимального напряжения; период, время и время смещения Экскурсия ответ vdence help Часть задачи должна поддерживаться соответствующим образом аннотированным Eraphical derstanding Don-echmica Hdlencz Z0cosh (x / 55) sinh «() m; Prnvide полностью объяснил slution; следующее; Значение 0i» # henx = [uto> siEnificamt fmures Значение x, когда 40 t0 3 значащих цифры Praice lully exuhine lerlphica Evidlence Hhcnt ‘ustify YOutAnawrrita uarA (в

г.

Рассмотрим схему, показанную на $ \ textbf {Рис.P30.71} $. Переключатель $ S $ замыкается в момент времени $ t = 0 $, вызывая ток $ i_1 $ через индуктивную ветвь и ток $ i_2 $ через емкостную ветвь. Начальный заряд конденсатора равен нулю, а заряд в момент времени t равен $ q_2 $. (a) Выведите выражения для $ i_1, i_2, $ и $ q_2 $ как функции времени. Выразите свои ответы в виде $ \ varepsilon, L, C, R_1, R_2, $ и $ t $. Для оставшейся части задачи пусть элементы схемы имеют следующие значения: $ \ varepsilon = 48 \, \ mathrm {V}, L = 8.0 \, \ mathrm {H}, C = 20 \, \ mu \, \ mathrm {F}, \ space R_1 = 25 \, \ Omega, $ и $ R_2 = 5000 \, \ Omega $. б) Каков начальный ток через индуктивную ветвь? Какой начальный ток через емкостную ветвь? (c) Каковы токи через индуктивную и емкостную ветви через долгое время после включения переключателя? Как долго это «долгое время»? Объяснять. (d) В какое время $ t_1 $ (с точностью до двух значащих цифр) токи $ i_1 $ и $ i_2 $ будут равны? ($ Подсказка: $ Вы можете рассмотреть возможность использования разложения в ряд для экспонент.) (e) Для условий, указанных в части (d), определите $ i_1 $. (е) Полный ток через батарею равен $ i = i_1 + i_2 $. В какое время $ t_2 $ (с точностью до двух значащих цифр) я буду равен половине его окончательного значения? ($ Подсказка: $ Работа с числами значительно упрощается, если делать подходящие приближения. Набросок $ i_1 $ и $ i_2 $ в зависимости от t может помочь вам решить, какие приближения являются действительными.)

Безопасность | Стеклянная дверь

Мы получаем подозрительную активность от вас или кого-то, кто пользуется вашей интернет-сетью.Подождите, пока мы подтвердим, что вы настоящий человек. Ваш контент появится в ближайшее время. Если вы продолжаете видеть это сообщение, напишите нам чтобы сообщить нам, что у вас возникли проблемы.

Nous aider à garder Glassdoor sécurisée

Nous avons reçu des activités suspectes venant de quelqu’un utilisant votre réseau internet. Подвеска Veuillez Patient que nous vérifions que vous êtes une vraie personne. Вотре содержание apparaîtra bientôt. Si vous continuez à voir ce message, veuillez envoyer un электронная почта à pour nous informer du désagrément.

Unterstützen Sie uns beim Schutz von Glassdoor

Wir haben einige verdächtige Aktivitäten von Ihnen oder von jemandem, der in ihrem Интернет-Netzwerk angemeldet ist, festgestellt. Bitte warten Sie, während wir überprüfen, ob Sie ein Mensch und kein Bot sind. Ihr Inhalt wird в Kürze angezeigt. Wenn Sie weiterhin diese Meldung erhalten, informieren Sie uns darüber bitte по электронной почте: .

We hebben verdachte activiteiten waargenomen op Glassdoor van iemand of iemand die uw internet netwerk deelt.Een momentje geduld totdat, мы узнали, что u daadwerkelijk een persoon bent. Uw bijdrage zal spoedig te zien zijn. Als u deze melding blijft zien, электронная почта: om ons te laten weten dat uw проблема zich nog steeds voordoet.

Hemos estado detectando actividad sospechosa tuya o de alguien con quien compare tu red de Internet. Эспера mientras verificamos que eres una persona real. Tu contenido se mostrará en breve. Si Continúas recibiendo este mensaje, envía un correo electrónico a para informarnos de que tienes problemas.

Hemos estado percibiendo actividad sospechosa de ti o de alguien con quien compare tu red de Internet. Эспера mientras verificamos que eres una persona real. Tu contenido se mostrará en breve. Si Continúas recibiendo este mensaje, envía un correo electrónico a para hacernos saber que estás teniendo problemas.

Temos Recebido algumas atividades suspeitas de voiceê ou de alguém que esteja usando a mesma rede. Aguarde enquanto confirmamos que Você é Uma Pessoa de Verdade.Сеу контексто апаресера эм бреве. Caso продолжить Recebendo esta mensagem, envie um email para пункт нет informar sobre o проблема.

Abbiamo notato alcune attività sospette da parte tua o di una persona che condivide la tua rete Internet. Attendi mentre verifichiamo Che sei una persona reale. Il tuo contenuto verrà visualizzato a breve. Secontini visualizzare questo messaggio, invia un’e-mail all’indirizzo per informarci del проблема.

Пожалуйста, включите куки и перезагрузите страницу.

Это автоматический процесс. Ваш браузер в ближайшее время перенаправит вас на запрошенный контент.

Подождите до 5 секунд…

Перенаправление…

Заводское обозначение: CF-102 / 6a41eef1eb794d90.

Произошла ошибка при настройке вашего пользовательского файла cookie

Произошла ошибка при настройке вашего пользовательского файла cookie

Этот сайт использует файлы cookie для повышения производительности. Если ваш браузер не принимает файлы cookie, вы не можете просматривать этот сайт.

Настройка вашего браузера на прием файлов cookie

Существует множество причин, по которым cookie не может быть установлен правильно. Ниже приведены наиболее частые причины:

  • В вашем браузере отключены файлы cookie. Вам необходимо сбросить настройки своего браузера, чтобы он принимал файлы cookie, или чтобы спросить вас, хотите ли вы принимать файлы cookie.
  • Ваш браузер спрашивает вас, хотите ли вы принимать файлы cookie, и вы отказались. Чтобы принять файлы cookie с этого сайта, используйте кнопку «Назад» и примите файлы cookie.
  • Ваш браузер не поддерживает файлы cookie. Если вы подозреваете это, попробуйте другой браузер.
  • Дата на вашем компьютере в прошлом. Если часы вашего компьютера показывают дату до 1 января 1970 г., браузер автоматически забудет файл cookie. Чтобы исправить это, установите правильное время и дату на своем компьютере.
  • Вы установили приложение, которое отслеживает или блокирует установку файлов cookie.Вы должны отключить приложение при входе в систему или проконсультироваться с системным администратором.

Почему этому сайту требуются файлы cookie?

Этот сайт использует файлы cookie для повышения производительности, запоминая, что вы вошли в систему, когда переходите со страницы на страницу. Чтобы предоставить доступ без файлов cookie потребует, чтобы сайт создавал новый сеанс для каждой посещаемой страницы, что замедляет работу системы до неприемлемого уровня.

Что сохраняется в файле cookie?

Этот сайт не хранит ничего, кроме автоматически сгенерированного идентификатора сеанса в cookie; никакая другая информация не фиксируется.

Как правило, в файлах cookie может храниться только информация, которую вы предоставляете, или выбор, который вы делаете при посещении веб-сайта. Например, сайт не может определить ваше имя электронной почты, пока вы не введете его. Разрешение веб-сайту создавать файлы cookie не дает этому или любому другому сайту доступа к остальной части вашего компьютера, и только сайт, который создал файл cookie, может его прочитать.

McCrary, Kim / Syllabus

Syllabus 7-го класса по математике

Описание курса: Этот класс предназначен не только для изучения математических понятий, но и для улучшения понимания учащимися математики и их способности решать задачи. Мы будем использовать несколько разных ресурсов. Студенты будут исследовать математические идеи, рассматривая реалистичные задачи, вместо того, чтобы смотреть только на числа. Некоторые проблемы связаны с реальными приложениями или необычными ситуациями, тогда как другие являются чисто математическими.Цель состоит в том, чтобы улучшить понимание учащимися и применение математических понятий. Ожидается, что студенты будут работать как самостоятельно, так и с другими.

Major 7 th класс по математике:

  • Анализировать пропорциональные отношения и использовать их для решения реальных и математических задач
  • Применять и расширять предыдущие представления об операциях с дробями для сложения, вычитания, умножения и деления рациональных чисел
  • Используйте свойства операций для создания эквивалентных выражений
  • Решать реальные и математические задачи, используя числовые и алгебраические выражения и уравнения
  • Нарисуйте, сконструируйте и опишите геометрические фигуры и опишите отношения между ними
  • Решение реальных и математических задач, связанных с измерением угла, площади, площади поверхности и объема
  • Использовать случайную выборку для вывода о совокупности
  • Сделайте неформальные сравнительные выводы о двух популяциях
  • Изучение случайных процессов, разработка, использование и оценка вероятностных моделей

Ожидаемые результаты в классе: ожидается студентов.. .

Be Cooperative

Будьте уважительны

Будьте ответственными

Будьте в безопасности

  • Следуйте указаниям
  • Участвовать в классе
  • Продолжайте работу
  • Будьте продуктивны
  • Разрешить конфликты надлежащим образом
  • Используйте подходящие слова, тон и громкость
  • Позаботьтесь о школьном имуществе
  • Забрать за собой
  • Будьте вежливы
  • Слушайте друг друга
  • Оставьте конструктивный отзыв
  • Довести материалы до класса
  • Правильно отображать идентификатор
  • Носить школьную форму
  • Выполнить задания и сдаться вовремя
  • Держать электронные устройства выключенными и убирать
  • Держите руки, ноги и другие предметы при себе
  • Сядьте правильно
  • Прогулка в классе и коридорах
  • Используйте школьные принадлежности в соответствии с инструкциями
  • Следуйте процедурам

Последствия: Последствия ненадлежащего поведения включают устное предупреждение, телефонный звонок родителям, задержание после школы / обеда, совещание учащихся и / или родителей и направление в офис.

Домашнее задание: Родители могут ожидать, что ученики будут выполнять домашнее задание каждый день. Исключения из нормы будут всякий раз, когда студенты проходят тестирование.

Поздняя работа: Классные и домашние задания являются обязательными, поэтому их следует сдавать. Родители будут уведомлены, когда учащиеся будут испытывать трудности с выполнением и / или постоянной отправкой домашних заданий.

Пропущенные работы: Студент обязан получить домашние задания, заметки и любую другую относящуюся к делу информацию, которая упущена во время отсутствия.Дополнительное время и полный кредит будут предоставлены для пропущенных заданий в результате отсутствия.

Оценка: Обновленные оценки можно просмотреть на родительском портале. Если у вас нет данных для входа в систему, свяжитесь с г-жой Томпсон, секретарем директора по телефону (314) 953-7500. Обратите внимание, что данные для входа в систему вашего ребенка (имя пользователя: номер студенческого номера, пароль: 6-значный код даты рождения) могут использоваться до тех пор, пока вы не настроите свою.

  • Итоговые задания — 70% от средней оценки учащегося.Сюда входят все тесты, викторины и проекты.
  • Формирующие задания — 30% от средней успеваемости учащегося. Сюда входят все домашние задания, классные работы и выходные листы.

Обратите внимание, что невыполненные задания будут оцениваться как ноль. Студентам предоставляется возможность сдать любые недостающие задания до конца двенадцатинедельного периода оценки.

Tardies: Ожидается, что учащиеся будут приходить в класс вовремя каждый день.Учащиеся будут отмечены как опоздавшие, если они явятся в класс после того, как время перехода закончилось и дверь класса была закрыта. Родителям будет сообщено о привычной опоздании. Кроме того, школьная политика будет соблюдаться в отношении дисциплины за опоздание.

Контактное лицо для родителей: Убедитесь, что ваша контактная информация остается актуальной. Если какая-либо информация требует изменения, пожалуйста, свяжитесь со школьным офисом по телефону (314) 953-7500. Обратите внимание, что с родителями свяжутся с позитивными новостями об образовательном росте их учеников или для обсуждения поведения и / или рабочих привычек, требующих внимания родителей.Обратите внимание, что вам предлагается общаться со мной, и я приветствую ваши телефонные звонки и электронные письма.

Сколько баллов вы можете получить за первую часть по математике Oge?

Сколько Оге наберет по математике в 2021 году?

Максимум количество баллов , которое может получить экзаменующийся за выполнение всей экзаменационной работы — 45 баллов … Таким образом, для сертификата достаточно набрать всего 13 б., Но для профиля c 2021 не менее 33 баллов … Рекомендуемая контрольная точка баллов для профильного класса — 28 баллов .

Сколько баллов по Оге по математике в 2021 году по 5?

« 5 »: 29-33, из них не менее 6 баллов за грамотность (по критериям ГК1-ГК4). Если по критериям ГК1-ГК4 ученик набрал меньше 6 баллов , ставится оценка «4.»

Какие темы нужно знать для ОГЭ по математике 2021?

Как проходит экзамен

  • Какие контрольные задания ОГЭ : Алгебра ; часть 1.
  • 1-5 Практические задания
  • 6 Числа и вычисления
  • 7 Неравенства и система координат
  • 8 Числа, вычисления, алгебраические выражения
  • 9 Уравнения и неравенства
  • 10 Статистика и теория вероятностей
  • 11 Функции и графики

Сколько Оге набрал 3 балла по математике в 2021 году?

Математика : по естественнонаучному профилю: 18 баллов , из которых не менее 6 по геометрии для экономического профиля: 18 баллов , из которых не менее 5 по геометрии по физическому математическому Профиль: 19 баллов , из них минимум 7 по геометрии

Сколько очков в Оге в России 2021?

Перевод баллов ОГЭ 2021 на Русский язык при оценке

29-33, из них не менее 6 баллов за грамотность (по критериям ГК1-ГК4).Если по критериям ГК1-ГК4 студент набрал менее 6 баллов , ставится оценка «4».

Сколько баллов ОГЭ вам нужно по математике?

по экономическому профилю: 18 баллов , из них не менее 5 по геометрии; по физическому математическому Профиль: 19 баллов , из которых не менее 7 по геометрии. Максимальная сумма баллов , которую участник может получить ОГЭ за выполнение всех экзаменационных работ, — 40 баллов .

Сколько баллов нужно в Русской Оге на 3?

23-28 баллов — оценка «4» (из них не менее 4 баллов за грамотность по критериям ГК1-ГК4). Если по критериям ГК1-ГК4 студент набрал менее 4 баллов , ставится оценка « 3 ». 29-33 баллов — оценка «5» (из них не менее 6 баллов за грамотность по критериям ГК1-ГК4).

Как оценивается ОГЭ по русскому языку?

Русский язык

4 и 5 классы по пятибалльной шкале с оценкой ОГЭ присуждаются при получении не менее четырех баллов при оценке 4 и не менее шести баллов при оценке 5 по критериям ГК-1 — ГК-4.Минимальный начальный балл, рекомендованный Рособрнадзором для отбора учеников в профильный класс, составляет 26 баллов.

Какие темы будут по математике?

Темы включены в GIA ( OGE ) по ставке математика

  • Числа и вычисления Натуральные числа …
  • Алгебраические выражения Литеральные выражения (переменные выражения) …
  • Уравнения и Неравенства Уравнения …
  • Функции и графики Числовые функции…
  • Координаты на прямой и плоскости Координатная линия …
  • Геометрия …
  • Статистика и теория вероятностей

Как лучше всего подготовиться к Оге по математике?

Как подготовиться к экзамену самостоятельно

  1. Составить план подготовки Лучше просто сделай это в определенной последовательности. …
  2. Повторить программу девятого класса Возьмите учебник 9 класса (авторы Макарычев, Миндюк, Нешков, Суворова) и еще раз пройдитесь по всем темам….
  3. Решите как можно больше задач

Пример заполнения ОГЭ Бланка на английском языке. Презентация «Правила заполнения Бланков ОГЭ». Не нужно переписывать условие задачи

Слайд 1.

Слайд 2.

Успех экзамена зависит не только от правильности ответов на задания, но и от правильности записи ответов в доплату ответов
Ой, я НГЕ сдавать не буду! Ведь я даже не знаю, как заполняются эти пробелы…
Встречайте инструкцию!
Разделы инструкции Общие положения Заполнение бланка ответов № 1 Заполнение бланка ответов № 2

Слайд 3.

Для выполнения экзаменационной работы участник организаторов GIA в аудитории дает из ответов
Форма ответов № 1 для выполнения тестов тестовой части (задание 2-14)
Форма ответов № 2 для записи ответов на задания с развернутым ответом
Дополнительный бланк ответа № 2

Слайд 4 .

Общие положения
Все ответы заполнены яркими черными чернилами. Допускается использование гелевых или капиллярных ручек. Участник тестирования должен изобразить каждую цифру и букву во всех заполненных полях бланка, тщательно копируя образец их написания из строки с образцами написания символов, расположенной вверху формы ответа № 1. Тщательное написание символов может привести к на то, что при автоматизированной обработке символ может быть распознан некорректно.Каждое поле в форме заполняется, начиная с первой позиции (включая поля для имени, фамилии, имени и патроната тестирования). Если у члена GIA нет информации для заполнения поля, он должен оставить его пустым (чтобы не стыковаться).

Слайд 5.

Общие положения
Строго запрещено: делать в полях форм, вне полей форм или в полях, заполненных типографским способом, любые записи и пометки, не связанные к содержанию пустых полей; Используйте для заполнения цветные ручки вместо черных, карандаш (даже для черновых записей на бланках), средства для исправления внесенных в информационную форму («мазок» и др.).

Слайд 6.

Заполнение формы ответа №1
В верхней (регистрационной) части Бланка №1 заполняются следующие поля (дублированная информация из регистрационной формы)
Перепишите с доски
Перевод от Ким
Переписать из документа ФИО. Футляр (серия — первые 4 цифры паспорта)
Заполняется организатором
Обязательно помещается в зарезервированное поле
р по адресу С.
SERGEE
ROMAN
А г о ре и ч
3 2 1 2
1 6 0 6 8 3
1 0 2 4
1 0 6
2 4
4 2 8 0 1 5
9 B.
GRUG Z.
0 3 0 6 1 5

Слайд 7.

Заполнение формы ответа №1
ПРИМЕЧАНИЕ Если по документу имя написано через дефис, то знак ставится в отдельная ячейка.
или
или
B erg — rozo in with to and y
A nna — m and ri
T Jemi P — Oolo B and H

Слайд 8.

Заполнение формы ответа №1
В области ответов нельзя допускать случайных отметок, клексов, полос размазанных чернил и т. д., поскольку при автоматизированной обработке это можно распознать как ответы на задачи Кима. Если вам не удалось избежать случайных отметок, их следует заменить в области «Замена ошибочных ответов» на те ответы, которые участник теста считает правильными. Краткий ответ записывается слева направо от номера задания, начиная с первой ячейки. Каждый символ записывается в отдельной ячейке. Любые скидки запрещены.
В средней части расположены поля для записи ответов на задания

Слайд 9.

Заполнение формы ответа №1
Если краткий ответ — фраза, первое слово записывается полностью, второе и все остальные слова записываются подряд без пробелов и других символов (сколько это будет)
последовательность чисел пишется без запятых
Если кратким ответом является слово, его нужно записать в той форме (род, число, падеж и т. д.), в которой оно стоит в предложении
3
Первая строка не заполнена
1
In zd about xnula
D erinny yu
Use in Shi sb
P and SKDETEY
We can non and h and tsi
2 3
1 2 5 6
4
6
1 7
3 9

Слайд 10.

Заполнение формы ответа №1
Примечания к ответам на задание 8
Не сохраняется дефисное написание Word. Например: Вдруг мелькнуло что-то мелькнуло среди кустов, росших здесь особенно густо.
Union не входит в грамматическую основу. Например: вот смотришь вокруг и думаешь.
Частица не входит в грамматическую основу. Например: старика никто не обижает, больную женщину …
Порядок записи основных членов приговора значения не имеет.Например: я видел медуллянт только поздно осенью.
или
H totomllknul около
G крышка и dum and y
N iktoneob and d and t
In in and Deli
I SAW

Slide 11.

Заполнение формы ответа №1
Изменить ответы В форме №1 ответа на задание необходимо поставить номер исправленного задания в соответствующие поля замены и записать новое значение правильного ответа на указанное задание.Если в поле замены ответов об ошибках выставлен номер такой же задачи, то будет учтено последнее исправление (сверху вниз и слева направо). Если в поле замены ответов об ошибках сделано только присвоение номера, будет считаться, что ответ на эту задачу не дан!
Внизу формы ответа №1 расположены поля для замены ошибочных ответов, предназначенные для исправления участниками своих ответов.

Слайд 12.

Заполнение формы ответа №1
Пример замены ответа на ошибку
4 V с n OMN и в
1 3 1 6 1 8

Слайд 13.

Заполнение формы ответа №2
Ответ Форма № 2 предназначена для записи ответов на задачи с развернутым ответом.
Информация для заполнения полей верхней части формы должна соответствовать информации, внесенной в форму ответа № 1
Основная часть формы занимает область записи ответов на задачи с ответом в развернутом виде. форма.На этом участке в границах участник записывает развернутые ответы на соответствующие задания (№1 и №15) строго в соответствии с требованиями КИМ.

Слайд 14.

Заполнение анкеты №2

1. Признание — это маленький талант прорастания, который может превратиться в могучее дерево, а также может засохнуть без тяжелого труда и хладнокровия. Считайте, что ваше признание — источник счастья. Я знаю много плохих профессионалов, равных молчаливых для своего дела, ненавижу его.Это происходит, если в юности была допущена ошибка при выборе профессии. Главное — любить свое дело, увлекаться им. Главное удовольствие в жизни — это творчество, потому что человек стремится и в процессе, и в результате получилось что-то красивое. Если бы я начал учиться у инженера, у врача, преподавателя, то я был бы просто человеком, зарабатывающим срочно на хлеб. Нужно, чтобы «Искра» вспыхнула в каждом. Тогда это будет реальный человек.
15.3. Что есть человечество? Большинство людей ответят на этот вопрос: это один из лучей качеств человека, побуждающего его помогать другим, эта способность поддержать в трудную минуту, позаботиться о старших.В тексте Б. Васильева мы видим, как нарушение этих простых и понятных правил человечности превращается в настоящую трагедию. Школьники для своего музея воруют у слепой женщины единство ее сына, погибшего на войне. Вот как она переживает письменное письмо: «Он поблек, умер, умер во-вторых, а теперь уже навсегда». Совсем недавно я прочитал о диком случае бесчеловечности в отношении дикой природы. В магазине женщину, пережившую блокаду, обвинили в том, что она ехала на повороте
. Изложение и эссе должны быть написаны обычным почерком (а не печатными буквами!), Но собраны и аккуратно строго на полях поля.
Не забудьте указать номер задачи: 1, 15.1, 15.2 или 15.3.
Из-за нехватки места для ответов на лицевой стороне формы ответов № 2 участник может продолжить запись на обратной стороне формы, сделав запись «Посмотрите на обратной стороне».
Между презентацией и рефератом необходимо сделать пропуск.

Слайд 15.

Заполнение формы ответа №2

На обратной стороне формы ответа №2 участник продолжает запись с того места работы, на которой она была прервана.
После того, как в форме № 2 для ответов осталось свободное место, участник должен поставить в этом месте британскую букву «Z», заполнив все свободное место.
уровень. Ее похитителя доставили в полицию, где женщина скончалась от стыда. В одном из комментариев я прочитал вопрос: «Человеческая жизнь равна трем масляным пакетам?» Там, где нет человечества, где нет понимания между людьми, человеческая жизнь перестает ничего стоить. Эти примеры говорят о том, что человек-вечность — это не просто красивое слово, это то качество, без которого мы, люди, мы просто не можем существовать, превратиться в злых животных, обреченных на самоуничтожение..
Прокладка формы бронирования № 2

Слайд 16.

Заполнение дополнительной формы ответов №2
При отсутствии места для ответов на основном письме ответов № 2, член GIA может продолжить запись на дополнительную оценку ответов №2, выданных организатором в аудитории по запросу участника в случае, когда не нашлось места для основной оценки ответов №2. В случае заполнения дополнительная форма ответов Нет.2 с развернутыми основными формами ответов № 2, ответы на дополнительные формы ответов № 2 не оцениваются.

Слайд 17.

Выполняйте задания правильно! Записывайте задачи правильно!
Удачи на экзамене!

Правила заполнения биланков ОГЭ Инструкция по заполнению бланков ответов участников ОГЭ

  • Для обработки экзаменационных материалов участников основного государственного экзамена (далее по тексту) используются следующие виды бланков:
  • бланк ответа № 1,
  • бланк ответа № 2,
  • дополнительный бланк ответа № 2.
  • Общая структура бланков едина для всех позиций.
  • Бланки — это машиночитаемые формы.
  • Все машиносчитываемые бланки заполнены яркими черными чернилами. Допускается использование только гелевых или капиллярных ручек.
  • Если у участника НГЭ нет информации для заполнения поля, он должен оставить его пустым (чтобы не стыковаться).
Правила заполнения бланков ответов участников экзамена
  • Внимание! Символы, введенные в бланк бланка резервуара, могут не распознаваться или неправильно распознаваться во время автоматизированной обработки бланков, что может существенно повлиять на результат рабочего теста.
Правила заполнения форм ответов участников экзамена
  • Участник ОГЭ должен изобразить каждую цифру и букву во всех заполненных полях оттока № 1, тщательно скопировав образец ее написания из ряда с образцами написание знаков, расположенных вверху бланка ответа № 1. Небрежное написание знаков может привести к тому, что при автоматизированной обработке символ может быть распознан некорректно.
  • Инструкция по заполнению форм ОГЭ размещена на сайте NIMRO в разделе GIA / Documents (http: // nimro.ru / gia / doc_ege.html)
Образцы формирования ответов участников ОГЭ
  • Область ответов на задачи с кратким ответом состоит из горизонтального ряда ячеек. Краткий ответ записывается справа от номера задания, начиная с первой ячейки. Рекомендуется сначала записать ответы на Черновик или на листы Кима, а затем перенести их в форму ответов №1.
  • В пробелах по каждой теме те поля, которые не следует заполнять.
Особенности оформления форм ответов участников ОГЭ
  • Форма краткого ответа устанавливается в виде КИМ по соответствующей тематике.
  • Образец инструкции Кима на русском языке:
Особенности оформления форм ответов участников ОГЭ
  • Пример инструкции Ким по математике:
  • Образец инструкций Ким по обществознанию:
Особенности оформления форм ответов участников ОГЭ
  • Если краткий ответ — фраза, то он записывается по следующему правилу: первое слово записывается полностью, второе и все остальные слова записываются в строка без пробелов и других символов (один символ в ячейке — столько символов войдет в поле настройки).
  • Если кратким ответом должно быть слово, пропущенное в каком-либо предложении, то это слово нужно записать в той форме (род, число, падеж и т. Д.), В которой оно должно стоять в предложении.
Особенности оформления бланков ответов участников ОГЭ
  • Если в коротком ответе требуется указать номера гостей и т. Д., Номера записываются в ряд без разделения пробелом и запятыми.
  • Ответом на настройку соответствия является число, полученное в таблице соответствия.
Особенности оформления форм ответов участников ОГЭ
  • Если числовой ответ получен в виде обыкновенной дроби, его следует перевести в десятичную дробь. В ответе, записанном в виде десятичной дроби, в качестве разделителя в отдельной ячейке следует указать запятую.
  • Если ответ получен в виде отрицательного числа, знак минус помещается в отдельную ячейку.
Особенности оформления форм ответов участников ОГЭ
  • Например, ответов на задания:
  • № 2: «Мне было больно, и я ушел.«
  • № 3: «Предложения, удовлетворяющие условию: 1,2,7»
  • запишется в форме в виде:
Особенности оформления форм ответов участников ОГЭ
  • В задаче № 4 по математике в ответе могут быть два корня квадратного уравнения, из которых нужно выбрать большее или меньшее (условие выбора — записано в КИМ):
Особенности оформления бланков отзывов участников ОГЭ Особенности оформления бланков отзывов участников ОГЭ
  • Форма ответа №2 предназначен для записи ответов на задания с развернутым ответом (строго в соответствии с требованиями инструкции к КИМ и к отдельным заданиям КИМ).
  • Форма представляет собой двустороннюю машиночитаемую форму и состоит из двух частей — верхней (зарегистрированное поле) и нижней.
  • Информация для заполнения верхней части формы ответа № 2 («Регион», «код темы» и «название темы», «номер варианта», «номер Ким») должна соответствовать информации, указанной в форме ответа № .1.
Особенности оформления бланков ответов участников ОГЭ Особенности оформления бланков ответов участников ОГЭ
  • Регистрация полей ответов № 2 не требуется
Особенности оформления форм ответов участников ОГЭ
  • Поле для ответов на задания с развернутым ответом расположено внизу формы ответа № 2, а также на обратной стороне формы. .
  • В этом поле участник НГЭ фиксирует развернутые ответы на соответствующие задания строго в соответствии с требованиями инструкции КИМ и отдельными заданиями Кима.
Особенности оформления бланков отзывов участников ОГЭ.
  • Все записи и рисунки на ответах № 2 выполнены черной гелевой или капиллярной ручкой, почерк отбитый.
  • При записи ответов необходимо соблюдать разметку листа (не выходя за разметку «в ячейку»), так как при сканировании символы, записанные в полях листа, будут «обрезаны».
Особенности оформления бланков ответов участников ОГЭ
  • При отсутствии места для ответов на лицевой стороне формы ответа No.2 участник ОГЭ должен продолжить запись на вращающейся стороне формы, сделав «Взгляд сзади» в нижней части области ответов.
Особенности оформления бланков отзывов участников ОГЭ
  • Важно! По иностранному языку, информатике и ИКТ, физике в форме ответа №2 в поле записи ответа необходимо написать: Код СИЗ, номер аудитории, № Ким (номер трад …) .
  • Это необходимо для проверки устной части (практическая часть, лабораторный эксперимент).
  • Образец заполнения Бланка в физике:
Образец оформления формы ответов № 2 участников НГЭ иностранный язык (устный)
  • СИЗ 3602.
  • Аудитория 22.
  • Номер Ким 1013546.
Образец дизайна формы ответов №2 участников ОГЭ по информатике и ИКТ
  • СИЗ 3602.
  • Аудитория 22.
  • Номер Ким 1013546.
Особенности оформления форм ответов участников ОГЭ
  • Если обе стороны формы ответа № 2 заполнены и требуется продолжить запись, то участнику выдается дополнительная форма ответа №2. Информация в дополнительном бланке ответа № 2 составляет у организаторов в аудитории при оформлении анкеты.
  • В случае заполнения дополнительной формы ответов № 2 с развернутыми основными формами ответов № 2, ответы на дополнительные формы ответов № 2 не оцениваются.
Примеры участников BNE
  • Благодарю за внимание
  • Теликова Ольга Петровна
  • научный сотрудник отдела сопровождения государственной (итоговой) аттестации
  • ГКУ НСО Нимо
  • [адрес электронной почты]

В целях обеспечения единых условий для всех участников ГИА-9 используются единые экзамены и обработка результатов.

По экзаменационным материалам НГЭ в составе:

1. Контрольные измерительные материалы (далее Ким), представляющие собой комплекты заданий стандартизированной формы;

2. Форма ответов № 1, предназначенная для регистрации члена ГИА-9 и дачи кратких ответов;

3. Форма ответов № 2 предназначена для развернутых ответов.

В связи с нехваткой места для ответов на основном письме ответов №2 участнику НГЭ необходимо продолжить запись на дополнительной сводной форме №2.2, выдается организатором в аудитории по запросу Участника НГЭ в случае, когда не нашлось места в зоне ответов основной формы ответа №2. При этом организаторы фиксируют соединение номеров основной и дополнительной форм ответов в специальных полях пропусков.

4. Дополнительный бланк ответов №2

Скачать формы ответов ОГЭ 2018 по всем предметам

Вещь Загрузить Blanches
Математика математика-Бланки-Оге
Форма ответа №2 (на все вопросы) бланк-Ответов-№2
Дополнительная форма ответа №2 dOP-BLANK-OTVETOV-№2
Русский язык Русский-Язик
Общественные науки общество.
Литература литература-Бланк.
История istoria-blanki.
Информатика информатика.
География geografia.
Физика физика-бланки.
Дополнительная форма ответа №2 Физика дополнения-Физика.
Биология biologia.
Химия Химия-Бланки.
Химия с экспериментом
Английский бланк угловой
Устный английский aNGL-USTN.
немецкий НЕМЕККИИ.
Немецкий устный НЕМЕЦКИИ-ЮСТН.
Испанский ИСПАНСКИЙ.
Устный испанский ИСПАНСКИЙ-УСТН.
французский francuzskii.
Французский устный francuzskii-USTN.
скачать

Формы GIA-9 — это машиночитаемые формы, которые подлежат автоматической обработке с помощью программного и аппаратного обеспечения. В процессе автоматизированной обработки формы, созданные в полях пропусков, информация с помощью программного обеспечения преобразуется в текст.

При небрежном заполнении форм с автоматической обработкой символы могут искажаться.

Правила заполнения биланков ОГЭ

общая часть

Слушатели ОГЭ выполняют экзаменационные работы по книгам ответов №1, формам ответов №2 и дополнительным пунктам ответов №2, правилам заполнения ниже. При заполнении форм НГЭ необходимо четко соблюдать правила заполнения пропусков, так как вводимая в формы информация сканируется и обрабатывается с помощью специального оборудования и программного обеспечения.

Все бланки демонов заполнены яркими черными чернилами.Допускается использование геля, капилляра или перьев. При отсутствии участника OGE указанные дескрипторы должны быть заменены дескриптором, соответствующим настоящим Правилам. Знак «Х» («Крест» в полях формы ответа № 1 (верхняя часть формы) не должен быть слишком толстым.

Если на ручке остается слишком толстая линия, то вместо креста в поле нужно провести только одну диагональ квадрата (любую). Участник НГЭ должен изобразить каждую цифру и букву во всех заполненных полях формы ответа №1, форма ответа №2 (верхняя часть) и дополнительная форма ответов №2 (верхняя часть) Тщательно копирующая свое написание своего написания из строки с образцами записи символов (см. Рис. 6), расположенных в верхней части форма ответа № 1.

Неосторожное написание символов может привести к тому, что при автоматизированной обработке символ может быть распознан некорректно.

Каждое поле формы заполняется, начиная с первой позиции, включая поля для записи имени, имени и отчества участника НГЭ.

Если у участника НГЭ нет информации для заполнения какого-то конкретного поля, то поле остается пустым (не для стыковки). При записи ответов в форму ответов №1 (средняя часть) необходимо строго следовать инструкции по выполнению работы (к группе задач и / или индивидуальным задачам), указанной в КИМ.

Не должно быть записей ответа на НГЭ Участнику НГЭ, содержащих информацию о его личностной форме №2. Например, ФИО, школа, название населенного пункта и т. Д.

Из-за незнания правил заполнения форм ОГЭ и восстановления ответов в них вы можете потерять свои драгоценные баллы, даже если ваши ответы верны.

Следовательно, необходимо четко разобраться, насколько правильно заполнять формы ответов НГЭ. Экспертиза экзамена будет проводиться в Бланках №1 и №2.

Форма ответа №1 в форме Русский язык бланка ответа № 2 единица для всех товаров

Форма ответа №1 предназначена для задач с кратким ответом.Он состоит из трех частей: верхней, средней и нижней. Регистрационные данные участника НГЭ вводятся в верхней части. В средней части ответы на часть 1 Ким. А нижняя часть предназначена для замены ошибочных ответов. Верхняя часть формы ответа №1 заполняется под руководством Организатора НГЭ в аудитории.

В форме ответов №2 развернуты ответы. Внизу этой формы вы можете прочитать сообщение:

Следовательно, для записи отрывных ответов необходимо использовать лицевую и вращающуюся стороны Blanca No.2. Дополнительно предоставляется дополнительная форма № 2. При необходимости вы можете потребовать это у Организатора НГЭ в его аудитории.


Общие правила заполнения бланков НГЭ 2020

Номер правила 1 . Писать можно только черной гелевой ручкой! Почему черный? Дело в том, что информация, которую вы ввели в форму №1, сканируется и распознается без участия человека. Компьютер переводит в символы все буквы, цифры и знаки на пробелах.Контуры букв, написанных шариком или гелевой ручкой синего или другого цвета, дают представление. Это мешает точному распознаванию написанной вами буквы, цифр или знака. В результате компьютер выдает ошибку. Также позаботьтесь о том, чтобы чернила на ручках не были слишком жирными и не оставляли за собой толстых линий. Будет обидно, если ваш правильный ответ будет засчитан как неправильный только потому, что вы использовали не ручку. Также следите за тем, чтобы не надевать заготовку цветка, это можно принять как свой ответ.Компьютер беспечности не прощает! Помнить!

Правило №2. Пишите только по образцу! Неактивное и нечеткое написание букв и цифр приведет к их неправильному распознаванию.

Правило № 3. Каждое поле в пробелах Заполните первой ячейкой! Если вы не знаете ответ на вопрос, то оставьте поле ответа пустым, не делайте этого. Также нельзя использовать раствор и ластик для исправления ответа. Вы можете исправить неправильный ответ в полях для замены ошибочных ответов.Первые две ячейки поля замены предназначены для записи номера задачи, ответ на которую нужно исправить. Запишите номер задания, а после тире правильный ответ. А что, если вы хотели исправить ответ задачи №20, а в поле замены №21 написали? Просто вычеркните 21, и в следующем поле замены напишите желаемый номер задачи и ответьте на него.

Правило №4. Запишите каждый символ в одну ячейку, не оставляя пробелов, даже если вы пишете фразы. Перед тем, как записывать ответ, внимательно прочтите инструкцию к тому заданию в Ким, которое вам нужно: написание чисел или слов.

Правило № 5. Не выходите за рамки форм № 2. Они тоже сканируют. Текст, который вышел за пределы, будет утерян. Эксперт проверяет только сканированные копии. Несоблюдение этого правила приведет к тому, что вашу работу будет сложно читать и правильно оценивать.

Правило № 6. Не делайте лишних знаков, даже точек, в поле или вне формы формы!

Как заполнить привязку ОГЭ 2020 на русском языке

    Во-первых, необходимо соблюдать общие правила, указанные на русском языке, для заполнения бланков ОГЭ на русском языке.Но есть некоторые особенности, которые необходимо знать и соблюдать.

  • Во-вторых, обратите внимание на то, что заполнение Бланка №1 в русском языке начинается не с первого поля, а со второго, так как ответ на первое задание написан Бланком №2.

    В-третьих, для правила №4 есть исключение. В каждом поле ответа всего 17 ячеек. Что делать, если в ответах писем больше? Согласно официальным источникам phipsis, в случае, когда в ответе более 17 символов и они не помещаются в каждую ячейку по одному, нужно выписывать ответ, не обращая внимания на ячейки.То есть не переворачивать свой ответ, а держать его в поле. Главное, писать поднимая трубку, чтобы персонажи в ответе не соприкасались друг с другом.

Обратите внимание на примечания к ответам к заданию 8.

  • поле ответа не передает дефис в слове. Например, словосочетание «Что-то упало» нужно написать так:
  • объединение не входит в грамматическую основу, поэтому они не пишут его в ответ.Например, сочетание слов «Посмотри и повтори» правильно напиши так:
  • частица не входит в грамматическую основу, записывается в поле ответа:
  • Порядок записи основных членов приговора может быть любым:
  • Презентация и эссе должны быть написаны на бланке ответа № 2 не печатными буквами, а обычным почерком, аккуратно и подбирая, не выходя за рамки.Не забываем указывать номера заданий, а между задачами есть небольшой отступ.

Описание презентации на отдельных слайдах:

1 слайд

Описание слайда:

Заполнение пробелов ОГЭ 2016 в русском языке и литературе Составитель Калашникова В.Н. Учитель русского языка и литературы МБОУ «Белоколодезские школы им. В.А. Даркова Шебекинского района Белгородской области» 2016

2 Слайд

Описание слайда:

Ответ Блан Нет.1 Для составления ответов тестовой части (Очередь 2-14) формы ответов № 2 для записи ответов на задания с развернутым ответом, дополнительная форма ответов № 2 Для выполнения экзаменационной работы участник ГИА организаторы в аудитории выдают ответы бланков

3 Слайд

Описание слайда:

Общие положения Все формы ответов заполнены яркими черными чернилами. Допускается использование гелевых или капиллярных ручек.Участник тестирования должен изобразить каждую цифру и букву во всех заполненных полях бланка, тщательно копируя образец их написания из строки с образцами написания символов, расположенной вверху формы ответа № 1. Тщательное написание символов может привести к на то, что при автоматизированной обработке символ может быть распознан некорректно. Каждое поле в форме заполняется, начиная с первой позиции (включая поля для имени, фамилии, имени и патроната тестирования). Если у члена GIA нет информации для заполнения поля, он должен оставить его пустым (чтобы не стыковаться).

4 Слайд

Описание слайда:

Общие положения категорически запрещены: — делать в полях форм, вне полей форм или в полях, заполненных типографским способом, любые записи и отметки, не относящиеся к содержанию листов форм; — Использовать цветные ручки для заполнения форм вместо черных, карандаша (даже для черновых записей на бланках), средств исправления внесенных в информационную форму («Масмазка» и др.).

5 Слайд

Описание слайда:

Заполнение анкеты №1 Код региона 31 Код организации образования 221308 Класс 9а код СИЗ 2132 Аудитория 8 Дата выполнения 03.06.2016 Код предмета 01 Название предмета Русский

6 Слайд

Описание слайда:

Перепишите из документа. ПОЛНОЕ ИМЯ. Дело (Серия — первые 4 цифры паспорта и номер) Заполнение формы ответа №1 Фамилия Михайлова Имя Алина Отчество Сергеевна Паспорт серии 1400 Номер 538917

7 Слайд

Описание слайда:

Заполнение формы ответа №1 Если согласно документу имя написано через дефис, то знак ставится в отдельную ячейку М и М и N _ С и Б и РКАЛЛА _ в и КТ \ u200bO P и I

8 Слайд

Описание слайда:

Первая строка не заполнена.Если кратким ответом является слово, то его нужно записать в той форме (род, число, падеж и т. Д.), В которой оно стоит в предложении. Если краткий ответ — фраза, то первое слово пишется полностью, второе и все остальные слова записываются подряд без пробелов и других символов (сколько это будет). Последовательность цифр пишется без запятых. (Внимательно читайте инструкцию к заданию !!!) Заполнение формы ответов №1 Посередине расположены поля для записи ответов на задания 1 2 5 6

9 Слайд

Описание слайда:

Примечания к ответам на задание 8 * Defis Writing Word не сохраняется.Например: Вдруг мелькнуло что-то мелькнуло среди кустов, росших здесь особенно густо. * Союз не входит в грамматическую основу. Например: вот смотришь вокруг и думаешь. * Частица не входит в грамматическую основу. Например: никто не обидится на старуху, на больную женщину … * Регистрация основных участников предложения значения не имеет. Например: я видел медуллянт только поздно осенью. или с a t o t o p r o l l k n u l o g l i d i d u m и n и k t o n e o b и d и t i in и d e l y in And d e l i

10 Слайд

Описание слайда:

Заполнение формы ответа №1 * В области ответов нельзя допускать случайных отметок, подтеков, размазанных чернил и т. Д., Так как при автоматизированной обработке это можно распознать как ответы на задания КИМ. * Если вам не удалось избежать случайных отметок, их следует заменить в области «Замена ошибочных ответов» на те ответы, которые участник теста считает правильными. * Краткий ответ записывается слева направо от номера задачи, начиная с первой ячейки. Каждый символ записывается в отдельной ячейке.* Любые сокращения запрещены.

11 Слайд

Описание слайда:

Заполнение формы ответа №1 Замена ошибочных ответов внизу формы ответа №1 Есть поля для замены ошибочных ответов, предназначенные для исправления участников в их ответах. Чтобы изменить ответы, сделанные в форме №1 ответа на задание, вам необходимо поставить номер исправленного задания в соответствующие поля замены и записать новое значение правильного ответа на указанное задание.Если в поле замены ответов об ошибках выставлен номер такой же задачи, то будет учтено последнее исправление (сверху вниз и слева направо). Если в поле замены ответов об ошибках сделано только присвоение номера, будет считаться, что ответ на эту задачу не дан!

12 Слайд

Описание слайда:

Заполнение формы ответа №2 формы ответа №2 предназначена для записи ответов на задачи с развернутым ответом. Информация Для заполнения полей верхней части формы необходимо соблюдать данные, указанные в ответе. Форма №1 Основная часть формы занимает область записи ответов на задачи с ответом в развернутой форме. В этой области, в пределах границ, участник фиксирует развернутые ответы на соответствующие задания (№1 и №15) строго в соответствии с требованиями Kim

.

13 Слайд

Описание слайда:

Заполнение формы ответа №2 Презентация и эссе должны быть написаны обычным почерком (не печатными буквами!), Но подбирая аккуратно и строго в пределах полей поля.Вы должны указать номер задачи: 1, 15.1, 15.2 или 15.3. Между презентацией и эссе необходимо сделать пропуск. При недостатке места для ответов на лицевой стороне формы ответов № 2 участник может продолжить запись на обратной стороне формы, сделав запись «Посмотрите на обратной стороне».

14 Слайд

Описание слайда:

Оборотная сторона анкеты №2 на оборотной стороне анкеты №2.2 Участник продолжает запись с того места, где она была прервана. При остатке свободного места на бланке ответов №2 организатор должен поставить в этом поле английскую букву «z», заполнив все свободное место.

15 Слайд

Описание слайда:

Заполнение дополнительной формы ответов №2 При отсутствии места для ответов по основному баллу Ответов №2 участнику НГЭ необходимо продолжить запись на дополнительном Бланке ответов №2.2, выдается организатором в аудитории по запросу участника НГЭ. В случае заполнения дополнительной формы ответов № 2 с развернутыми основными формами ответов № 2, ответы на дополнительные формы ответов № 2 не оцениваются.

Author: alexxlab

Добавить комментарий

Ваш адрес email не будет опубликован. Обязательные поля помечены *